Vous êtes sur la page 1sur 62

Université Moulay Ismaïl

Faculté des Sciences et Techniques


Département de Mathématiques

Filière : Tronc commun MIP

Module : M135

=======================================================

ANALYSE 3 :
Fonctions de plusieurs variables
et calcul des intégrales multiples

Exercices et examens corrigés


=======================================================

Professeur : S. M. DOUIRI

Année universitaire : 2021/2022


Université Moulay Ismaïl
Faculté des Sciences et Techniques
Département de Mathématiques

Filière : Tronc commun MIP


S.
Module : M135
M
=======================================================

ANALYSE 3 :
.D
Fonctions de plusieurs variables
et calcul des intégrales multiples

Exercices et examens corrigés


=======================================================
O

Professeur : S. M. DOUIRI
UI
RI
Année universitaire : 2021/2022

1
Table des matières
S.
1 Notions Topologiques dans Rn 3

2 Fonctions de plusieurs variables réelles 18


M
3 Calculs des intégrales doubles et triples 31

4 Correction de certains examens 42


4.1 Examen d’Analyse III (02 Mars 2021) . . . . . . . . . . . . . . . . . . . . . . . . 42
4.2 Examen d’Analyse III (11 Mars 2020) . . . . . . . . . . . . . . . . . . . . . . . . 44
.D
4.3 Examen d’Analyse III (13 Janvier 2020) . . . . . . . . . . . . . . . . . . . . . . 46
4.4 Examen d’Analyse III (07 Janvier 2019) . . . . . . . . . . . . . . . . . . . . . . 52
O
UI
RI

2
Chapitre 1
Notions Topologiques dans Rn
S.
Exercice 1.0.1. Pour deux éléments x = (x1 , . . . , xn ) et y = (y1 , . . . , yn ) de Rn , on définit le
produit scalaire de x et y par
n
X
M
(x|y) = x i yi .
i=1

1. Montrer que (x|x)λ2 + 2λ(x|y) + (y|y) ≥ 0 pour tout λ ∈ R.


2. Déduire l’inégalité de Cauchy-Schwarz suivante :
.D
(x|y)2 ≤ (x|x) (y|y) ∀x, y ∈ Rn .

Solution. 1. Pour tout λ ∈ R, on a


n
! n n
X X X
(x|x)λ + 2λ(x|y) + (y|y) =
2
x2i λ2 + 2λ xi y i + yi2
i=1 i=1 i=1
n 
X 
= λ2 x2i + 2λ xi yi + yi2
O
i=1
n
X
= (λ xi + yi )2 ≥ 0
i=1
UI
2. Pour tous éléments x et y fixés dans Rn , le polynôme P (λ) = (x|x)λ2 + 2λ(x|y) + (y|y),
dont le degré est 2, est toujours positif pour tout λ ∈ R. Alors son discriminant ∆ est
négatif, c’est-à-dire, ∆ = 4(x|y)2 − 4(x|x)(y|y) ≤ 0. D’où le résultat de Cauchy-Schwarz.

Exercice 1.0.2. 1. Montrer que || ||1 , || ||2 et || ||∞ sont des normes sur Rn .
2. Plus généralement, pour p ∈ [1, +∞[ et x = (x1 , . . . , xn ) ∈ Rn on pose
RI
n
!1
X p
kxkp = |xi | p
.
i=1

Montrer que k kp définit une norme sur Rn .


Indication : Utiliser l’inégalité de Minkowski suivante
1 1 1
(|x1 + y1 |p + · · · + |xn + yn |p ) p ≤ (|x1 |p + · · · + |xn |p ) p + (|y1 |p + · · · + |yn |p ) p

pour tous réels x1 , . . . , xn , y1 , . . . , yn .

3
S.M. DOUIRI

3. Montrer que pour tout p ∈ [1, +∞[ les normes k kp et k k∞ sont équivalentes.
4. Déduire que pour tout x ∈ Rn on a

lim kxkp = kxk∞ .


p→∞

n
P
Solution. 1. F L’application k k1 : x 7→ kxk1 = |xi | est bien définie sur Rn à valeurs
i=1
dans R+ . Vérifie-t-elle les 3 propriétés d’une norme ?
i) La séparation : Pour tout x = (x1 , . . . , xn ) ∈ Rn ,
n
P
kxk1 = 0 ⇔ |xi | = 0 ⇔ |xi | = 0, ∀i ∈ {1, . . . , n}
S.
i=1
⇔ xi = 0, ∀i ∈ {1, . . . , n} ⇔ x = 0Rn .

ii) L’homogénéité : Pour tous x = (x1 , . . . , xn ) ∈ Rn et λ ∈ R on a


M
n
X n
X n
X
kλxk1 = |λxi | = |λ| |xi | = |λ| |xi | = |λ|kxk1 .
i=1 i=1 i=1

iii) L’inégalité triangulaire : Pour tous x = (x1 , . . . , xn ) ∈ Rn et y = (y1 , . . . , yn ) ∈ Rn ,


on a
.D
n
P
kx + yk1 = |xi + yi |
i=1
n
P
≤ (|xi | + |yi |)
i=1
n
P Pn
≤ |xi | + i=1 |yi |
i=1
kx + yk1 ≤ kxk1 + kyk1 .
O
Donc, k k1 est une norme sur Rn .
 n 1
P 2
F L’application k k2 définie sur R par kxk2 = n
x2i est positive. Vérifie-t-elle les 3
i=1
propriétés d’une norme ?
UI
i) La séparation : Pour tout x = (x1 , . . . , xn ) ∈ Rn ,
 n 1 n
P 2 P
kxk2 = 0 ⇔ x2i =0⇔ x2i = 0 ⇔ x2i = 0, ∀i ∈ {1, . . . , n}
i=1 i=1
⇔ xi = 0, ∀i ∈ {1, . . . , n} ⇔ x = 0Rn .
RI
ii) L’homogénéité : Pour tous x = (x1 , . . . , xn ) ∈ Rn et λ ∈ R on a
 n 1  n 1  n
1
P 2 P 2 P 2
kλxk2 = (λxi ) 2
= λ2 x2i = λ 2
x2i
i=1 i=1 i=1
 n 1
P 2
= |λ| x2i = |λ|kxk2 .
i=1

iii) L’inégalité triangulaire : En utilisant l’inégalité de Cauchy-Schwarz suivante "à vérifier" :

< x|y >2 ≤< x|x > < y|y > ∀x = (x1 , . . . , xn ) ∈ Rn , ∀y = (y1 , . . . , yn ) ∈ Rn ,

4
S.M. DOUIRI
n
P
où < x|y >= xi yi est le produit scalaire entre x et y. En particulier,
i=1

n
X n
X
< x|x >= x2i = kxk22 et < y|y >= yi2 = kyk22 ,
i=1 i=1

donc, l’inégalité de Cauchy-Schwarz est équivalente à

| < x|y > | ≤ kxk2 kyk2 .

Alors, pour tous x = (x1 , . . . , xn ) ∈ Rn et y = (y1 , . . . , yn ) ∈ Rn , on a


n
P n
P n
P n
P n
P
kx + yk22 = (xi + yi )2 = (x2i + 2xi yi + yi2 ) = x2i + 2 xi y i + y2
S.
i
i=1 i=1 i=1 i=1 i=1
= + 2 < x|y >
kxk22 ≤ + 2| < x|y > | +
+kyk22 kxk22 kyk22
≤ + 2kxk2 kyk2 + kyk2 (D’après l’inégalité de Cauchy-Schwarz)
kxk22 2

kx + yk2 ≤ (kxk2 + kyk2 )2


2
M
Ce qui implique l’inégalité triangulaire

kx + yk2 ≤ kxk2 + kyk2 , ∀x ∈ Rn , ∀y ∈ Rn .


.D
Finalement, l’application k k2 vérifie bien les 3 conditions d’une norme. Donc, k k2 est une
norme sur Rn .
F L’application k k∞ définie sur Rn par kxk∞ = sup |xi | est positive. Vérifie-t-elle les 3
i=1,...,n
propriétés d’une norme ?
i) La séparation : Pour tout x = (x1 , . . . , xn ) ∈ Rn ,

kxk∞ = 0 ⇔ sup |xi | = 0 ⇔ |xi | = 0, ∀i ∈ {1, . . . , n}


O
i=1,...,n
⇔ xi = 0, ∀i ∈ {1, . . . , n} ⇔ x = 0Rn .

ii) L’homogénéité : Pour tous x = (x1 , . . . , xn ) ∈ Rn et λ ∈ R on a


UI
kλxk∞ = sup |λxi | = sup |λ| |xi | = |λ| sup |xi | = |λ|kxk∞ .
i=1,...,n i=1,...,n i=1,...,n

iii) L’inégalité triangulaire : Pour tous x = (x1 , . . . , xn ) ∈ Rn et y = (y1 , . . . , yn ) ∈ Rn , on a


RI
kx + yk∞ = sup |xi + yi |
i=1,...,n
≤ sup (|xi | + |yi |)
i=1,...,n
≤ sup |xi | + sup |yi |
i=1,...,n i=1,...,n
kx + yk∞ ≤ kxk∞ + kyk∞ .

Donc, k k∞ est une norme sur Rn .


Pn 1
2. Par définition, l’application k kp définie sur Rn par kxkp = ( i=1 |xi |p ) p est positive.
Vérifie-t-elle les 3 propriétés d’une norme ?

5
S.M. DOUIRI

i) La séparation : Pour tout x = (x1 , . . . , xn ) ∈ Rn ,

n
!1 n
X p X
kxkp = 0 ⇔ |xi |p =0⇔ |xi |p = 0
i=1 i=1
⇔ |xi | = 0, ∀i ∈ {1, . . . , n} ⇔ xi = 0, ∀i ∈ {1, . . . , n}
⇔ x = 0Rn .

ii) L’homogénéité : Pour tous x = (x1 , . . . , xn ) ∈ Rn et λ ∈ R on a

n
!1 n
!1
X p X p
kλxkp = |λxi | p
= p
|λ| |xi | p
S.
i=1 i=1
n
!1 n
!1
X p X p
= |λ| p
|xi |p
= |λ| p
|xi |
i=1 i=1
= |λ|kxkp .
M
iii) L’inégalité triangulaire : Pour tous x = (x1 , . . . , xn ) ∈ Rn et y = (y1 , . . . , yn ) ∈ Rn , on a

n
!1
X p 1
kx + ykp = |xi + yi | p
= (|x1 + y1 |p + |x2 + y2 |p + · · · + |xn + yn |p ) p
.D
i=1
1 1
≤ (|x1 | + |x2 |p + · · · + |xn |p ) p + (|y1 |p + |y2 |p + · · · + |yn |p ) p (Inég. de Minkowoski),
p

n
!1 n
!1
X p X p
= |xi |p
+ |yi | p
= kxkp + kykp .
i=1 i=1

Finalement, l’application k kp vérifie bien les 3 conditions d’une norme. Donc, k kp est une
O
norme sur Rn .
3. Soit p ∈ [1, +∞[. Montrons que les deux normes k kp et k k∞ sont équivalentes, c’est-à-dire
qu’il existe α > 0 et β > 0 tels que
UI
αkxk∞ ≤ kxkp ≤ βkxk∞ , ∀x ∈ Rn .

Soit x ∈ Rn , il existe i0 ∈ {1, . . . , n} tel que

kxk∞ = sup |xi | = sup(|x1 |, |x2 |, . . . , |xn |) = |xi0 |.


i∈{1,...,n}
RI
n
P
On a |xi0 |p ≤ |xi |p , alors
i=1

n
!1
1 X p
kxk∞ = |xi0 | = (|xi0 |p ) ≤ p |xi |p .
i=1

D’où kxk∞ ≤ kxkp , et par suite on peut choisir α = 1.


Maintenant, pour tout i ∈ {1, . . . , n} on a |xi | ≤ kxk∞ . Alors,

|xi |p ≤ kxkp∞ , ∀i ∈ {1, . . . , n}.

6
S.M. DOUIRI

Donc,
n
X n
X
|xi |p ≤ kxkp∞ ,
i=1 i=1

c’est-à-dire,
n
X
|xi |p ≤ nkxkp∞ ,
i=1

ce qui implique que


n
!1
X p 1
|xi | p
≤ (nkxkp∞ ) p .
i=1
1 1
Par conséquence, kxkp ≤ n p kxk∞ . Alors, on peut prendre β = n p . Finalement, on a
S.
1
kxk∞ ≤ kxkp ≤ n p kxk∞ , ∀x ∈ Rn .

Ce qui signifie que les deux normes k kp et k k∞ sont équivalentes.


M
4. Soit x ∈ Rn . D’après la question précédente, on a
1
kxk∞ ≤ kxkp ≤ n p kxk∞ , ∀p ∈ [1, +∞[.
Par passage à la limite, on obtient
.D
 1 
lim kxk∞ ≤ lim kxkp ≤ lim n p kxk∞ .
p→∞ p→∞ p→∞

   
1 1
Or lim kxk∞ = kxk∞ et lim n p kxk∞ = lim n p kxk∞ = kxk∞ puisque
p→∞ p→∞ p→∞
1
lim n =
p→∞
p lim exp( p1
p→∞
ln n) = 1. Par suite,
O
lim kxkp = kxk∞ .
p→∞

Exercice 1.0.3. Soit k k une norme sur Rn .


1. Vérifier que l’application d définie sur Rn × Rn par d(x, y) = kx − yk est une distance sur
UI
Rn (la distance d est dite associée à la norme k k).
q
2. Montrer que l’application d0 définie sur Rn × Rn par d0 (x, y) = ||x − y|| est une distance
sur Rn . Est-elle associée à une norme ?
3. Soit N une norme sur R. Montrer qu’il existe α > 0 tel que pour tout x ∈ R on a
N (x) = α |x|.
RI
Solution. 1. L’application d définie sur Rn × Rn par d(x, y) = kx − yk est une application
positive bien définie. Vérifie-t-elle les 3 propriétés d’une distance ?
i) La séparation : Pour tous x = (x1 , . . . , xn ) ∈ Rn et y = (y1 , . . . , yn ) ∈ Rn on a

d(x, y) = 0 ⇔ kx − yk = 0 ⇔ x − y = 0Rn ⇔ x = y.

ii) La symétrie : Pour tous x = (x1 , . . . , xn ) ∈ Rn et y = (y1 , . . . , yn ) ∈ Rn on a

d(x, y) = kx − yk = k(−1)(y − x)k = | − 1|ky − xk = ky − xk = d(y, x)

7
S.M. DOUIRI

iii) L’inégalité triangulaire : Pour tous x = (x1 , . . . , xn ) ∈ Rn , y = (y1 , . . . , yn ) ∈


Rn et z = (z1 , . . . , zn ) ∈ Rn on a

d(x, y) = kx − yk = kx − z + z − yk
≤ kx − zk + kz − yk
d(x, y) ≤ d(x, z) + d(z, y)

Donc, d est une distance sur Rn .


q
2. Par définition, l’application d0 définie sur Rn × Rn par d0 (x, y) = d(x, y) est positive.
vérifie-elle les 3 propriétés d’une distance ?
q
i) La séparation : ∀(x, y) ∈ Rn ×Rn , d0 (x, y) = 0 ⇔ d(x, y) = 0 ⇔ d(x, y) = 0 ⇔ x = y,
puisque d est une distance.
S.
q q
ii) La symétrie : ∀(x, y) ∈ Rn × Rn , d0 (x, y) = d(x, y) = d(y, x) = d0 (y, x).
iii) L’inégalité triangulaire : Pour tout (x, y, z) ∈ Rn × Rn × Rn et puisque d est √ une
distance, on a d(x, y) q q y). Or la fonction usuelle de la racine t √
≤ d(x, z)+d(z, 7→ t est
M
croissante, alors on a d(x, y) ≤ d(x, z) + d(z, y). En utilisant le fait que t + t0 ≤
√ √ q q q
t + t0 (à vérifier), on déduit que d(x, z) + d(z, y) ≤ d(x, z) + d(z, y). Par
q q q
conséquence, d(x, y) ≤ d(x, z) + d(z, y). D’où d0 (x, y) ≤ d0 (x, z) + d0 (z, y).
Finalement, l’application d0 vérifie bien les 3 conditions d’une distance. Donc, d0 est une
.D
distance sur Rn .
Supposons que cette distance d0 est induite par une norme N sur Rn , c’est-à-dire qu’il
existe une norme N telle que pour tous x, y ∈ Rn , d0 (x, y) = N (x − y). En particulier,
pour tout x ∈ Rn \ {0} on a d0 (x, 0) = N (x). Ce qui entraine que
√ q q q q
2 ||x|| = 2||x|| = ||2x|| = d0 (2x, 0) = N (2x) = 2N (x) = 2d0 (x, 0) = 2 ||x||,
O
qui est une contradiction puisque le vecteur x est non nul. Par conséquent, la distance d0
n’est associée à aucune norme sur Rn .
3. Soit N une norme sur R, alors pour tout x ∈ R on a
UI
N (x) = N (x × 1) = |x| N (1) = α |x|, où α = N (1) > 0.

Exercice 1.0.4. Soient a et b deux réels strictement positifs. On pose


s
x2 y 2
N (x, y) = + 2, ∀(x, y) ∈ R2 .
RI
a2 b

1. Montrer que N est une norme sur R2 .


2. Prouver que les normes N et || ||2 sont équivalentes.
3. Déterminer et dessiner les boules fermées unitées pour les normes N et || ||2 .
q
2
Solution.
2
1. Pour tout X = (x, y) ∈ R on a N (X) = N (x, y) = xa2 + yb2 ≥ 0. Reste à
2

montrer que l’application N satisfait les 3 propriétés d’une norme sur R2 . Pour simplifier
x y
les calculs, on peut remarquer que N (x, y) = k( , )k2 et on utilise le fait que k k2 est
a b
une norme.

8
S.M. DOUIRI

i) La séparation : Pour tout (x, y) ∈ R2 , on a


s
x2 y 2 x y
N (x, y) = 0 ⇔ 2
+ 2 = 0 ⇔ k( , )k2 = 0
a b a b
x y x y
⇔ ( , ) = (0, 0) ⇔ = = 0
a b a b
⇔ x = y = 0 ⇔ (x, y) = (0, 0) = 0R2

ii) L’homogénéité : Pour tous X = (x, y) ∈ R2 et λ ∈ R on a

λx λy
N (λX) = N (λ(x, y)) = N (λx, λy) = k( , )k2
a b
S.
x y x y
= kλ( , )k2 = |λ|k( , )k2 = |λ|N (x, y) = |λ|N (X).
a b a b
Alors N (λX) = |λ|X, ∀X ∈ R2 .
iii) L’inégalité triangulaire : Pour tous X = (x, y) ∈ R2 et X 0 = (x0 , y 0 ) ∈ R2 , on a
M
N (X + X 0 ) = N ((x, y) + (x0 , y 0 )) = N (x + x0 , y + y 0 )
x + x0 y + y 0 x y x0 y 0
= k( , )k2 = k( , ) + ( , )k2
a b a b a b
x0 y 0
.D
x y
≤ k( , )k2 + k( , )k2
a b a b
= N (x, y) + N (x , y )
0 0

= N (X) + N (X 0 ).

Finalement, l’application N vérifie bien les 3 conditions d’une norme. Donc, N est une
norme sur R2 .
O
2. Posons α = max(a, b) et β = min(a, b), alors
1 1 1 1 1 1
≤ ≤ et ≤ ≤ ,
α a β α b β
UI
ce qui entraine que
x2 x2 x2 y2 y2 y2
≤ ≤ et ≤ ≤ , ∀(x, y) ∈ R2 .
α2 a2 β2 α2 b2 β2
Par sommation terme à terme on obtient
RI
x2 + y 2 x2 y 2 x2 + y 2
2
≤ 2
+ 2
≤ 2
, ∀(x, y) ∈ R2
α a b β
et par suite s s s
x2 + y 2 x2 y 2 x2 + y 2
≤ + 2 ≤ , ∀(x, y) ∈ R2
α2 a2 b β2
c’est-à-dire,
1 1
k(x, y)k2 ≤ N (x, y) ≤ k(x, y)k2 , ∀(x, y) ∈ R2 .
α β
Ce qui signifie que les deux normes N et || ||2 sont équivalentes.

9
S.M. DOUIRI

3. Pour la norme k k2 , la boule fermée unité est Bk0 k2 (0R2 , 1) et on a

Bk0 k2 (0R2 , 1) = Bk0 k2 ((0, 0), 1) = {(x, y) ∈ R2 | ||(x, y)||2 ≤ 1}


q
2
= {(x, y) ∈ R | x2 + y 2 ≤ 1}
= {(x, y) ∈ R2 | x2 + y 2 ≤ 1}

qui représente le disque de centre (0, 0) et de rayon 1.


S.
M

Figure 1.1 – Boule fermée unité pour la norme k k2


.D
Pour la norme N, la boule fermée unité est BN
0
(0R2 , 1) et on a
0
BN (0R2 , 1) = BN
0
((0, 0), 1) = {(x, y) ∈ R2 | N (x, y) ≤ 1}
s
2 x2 y 2
= {(x, y) ∈ R | + 2 ≤ 1}
a2 b
O
2 2
x y
= {(x, y) ∈ R2 | 2 + 2 ≤ 1},
a b
qui représente l’ellipse de centre (0, 0) et de demi-axes a et b.
UI
RI

Figure 1.2 – Boule fermée unité pour la norme N

10
S.M. DOUIRI

Exercice 1.0.5. Soient N1 et N2 deux normes équivalentes sur Rn .


1. Prouver que N1 et N2 définissent la même topologie sur Rn , c’est-à-dire, la notion d’ouvert
ne change pas entre les deux normes.
2. Même question pour la convergence d’une suite (xk )k∈N dans Rn .
Solution. 1. N1 et N2 sont deux normes équivalentes sur l’espace vectoriel normé Rn , alors
il existe deux réels strictement positifs α > 0 et β > 0 tels que

αN1 (x) ≤ N2 (x) ≤ βN1 (x), ∀x ∈ Rn .

Soit θ un ouvert de Rn pour la norme N1 , est-t-il ouvert pour la norme N2 ?


Soit a ∈ θ, alors il existe r > 0 tel que BN1 (a, r) ⊂ θ. Pour tout x ∈ BN2 (a, α r), on a
N2 (x − a) < α r. D’après l’équivalence entre N1 et N2 , on a α N1 (x − a) ≤ N2 (x − a) < α r,
S.
ce qui implique que N1 (x − a) < r, c’est-à-dire, x ∈ BN1 (a, r). Par suite BN2 (a, α r) ⊂
BN1 (a, r) ⊂ θ. D’où θ reste ouvert dans Rn pour la norme N2 .
Réciproquement, Soit θ0 un ouvert de Rn pour la norme N2 , est-t-il ouvert pour la norme
N1 ?
M
Soit a ∈ θ0 , alors il existe r0 > 0 tel que BN2 (a, r0 ) ⊂ θ0 . Pour tout x ∈ BN1 (a, rβ ), on a
0

N1 (x − a) < rβ . D’après l’équivalence entre N1 et N2 , on a N2 (x − a) ≤ βN1 (x − a) < β rβ ,


0 0

ce qui implique que N2 (x − a) < r0 , c’est-à-dire, x ∈ BN2 (a, r0 ). Par suite BN1 (a, rβ ) ⊂
0

BN2 (a, r0 ) ⊂ θ0 . D’où θ0 reste ouvert dans Rn pour la norme N1 .


.D
2. Soit (xk )k∈N une suite de Rn qui converge vers l pour la norme N1 , alors
ε
∀ε > 0, ∃k0 ∈ N, ∀k ≥ k0 on a N1 (xk − l) < .
β
Ce qui implique que

∀ε > 0, ∃k0 ∈ N, ∀k ≥ k0 on a βN1 (xk − l) < ε.


O
D’après l’équivalence entre les deux normes N1 et N2 , on a N2 (xk − l) ≤ βN1 (xk − l) et
par suite
∀ε > 0, ∃k0 ∈ N, ∀k ≥ k0 on a N2 (xk − l) < ε.
UI
Ce qui signifie que la suite (xk )k∈N converge aussi vers l pour la norme N2 .
Réciproquement, Soit (xk )k∈N une suite de Rn qui converge vers l pour la norme N2 , alors

∀ε > 0, ∃k0 ∈ N, ∀k ≥ k0 on a N2 (xk − l) < αε.

D’après l’équivalence entre les deux normes N1 et N2 , on a αN1 (xk − l) ≤ N2 (xk − l) et


RI
par suite
∀ε > 0, ∃k0 ∈ N, ∀k ≥ k0 on a αN1 (xk − l) < αε.
c’est-à-dire,
∀ε > 0, ∃k0 ∈ N, ∀k ≥ k0 on a N1 (xk − l) < ε.
Ce qui signifie que la suite (xk )k∈N converge aussi vers l pour la norme N1 . D’où la notion
de la convergence ne change pas si on change une norme par une autre norme équivalente.
Exercice 1.0.6. 1. Montrer que toute Sphère de Rn est un fermé. Est-elle compacte ?
2. Prouver que toute partie finie A de Rn est fermée. Est-elle compacte ?

11
S.M. DOUIRI

Solution.
1. Soit S(a, r) une sphère de Rn pour une norme k k, c’est-à-dire,
S(a, r) = {x ∈ Rn /kx − ak = r}
= {x ∈ Rn /kx − ak ≥ r et kx − ak ≤ r}
= {x ∈ Rn /kx − ak ≤ r} ∩ {x ∈ Rn /kx − ak ≥ r}
= B 0 (a, r) ∩ {x ∈ Rn /kx − ak < r}C
= B 0 (a, r) ∩ {Rn
B(a,r)

S(a, r) = B 0 (a, r)\B(a, r)


On a la boule ouverte B(a, r) est un ouvert, alors son complémentaire {Rn est un fermé.
B(a,r)

Comme la boule fermée B 0 (a, r) est un fermé, alors B 0 (a, r) ∩ {Rn est aussi un fermé de
B(a,r)

Rn . Par conséquence, la sphère S(a, r) est fermée dans Rn . Est-elle compacte ? En effet,
pour tout x ∈ S(a, r) on a kx − ak = r. Or, |kxk − kak| ≤ kx − ak, alors kxk ≤ r + kak.
S.
D’où, il existe M > 0 (on peut choisir M = r + kak) tel que pour tout x ∈ S(a, r) on a
kxk ≤ M, c’est-à-dire, la sphère S(a, r) est bornée. Par conséquent, elle est compacte.
M
.D

Figure 1.3 – Schéma explicative pour l’espace R2


O
2. Soit A = {a1 , a2 , . . . , ap } une partie finie de Rn , alors A = ∪pi=1 {ai }. Il suffit de montrer
que chaque singleton {ai } est fermé. Soit {a} un singleton de Rn , pour tout x ∈ {a}C , on
UI
a x 6= a, c’est-à-dire, kx − ak > 0, donc il existe ε tel que 0 < ε < kx − ak. Montrons que
B(x, ε) ⊂ {a}C . En effet, soit y ∈ B(x, ε), alors kx−yk < ε. Or, kx−ak ≤ kx−yk+ky−ak,
alors

ky − ak ≥ kx − ak − kx − yk
RI
> kx − ak − ε
>0
D’où y 6= a, c’est-à-dire, y ∈ {a}C . Par conséquence, {a}C est un ouvert, ce qui signifie
que le singleton {a} est un fermé, et par suite, toute partie fine est fermé comme réunion
finie des singletons. Reste à montrer que la partie finie A est bornée pour déduire sa
compacité. Soit M = sup kai k, alors pour tout point ai de A on a kai k ≤ M, donc la
i=1,...,p
partie A est bornée, et par suite elle est compacte.
Exercice 1.0.7. 1. Montrer que toute boule ouverte (resp. boule fermée) dans Rn est un
ouvert (resp. fermé).

12
S.M. DOUIRI
S.
Figure 1.4 – Schéma explicative pour l’espace R2

2. Déterminer la nature topologique (ouvert, fermé ou compact) des ensembles :


M
A = {(x, y) ∈ R2 /|x| + |y| ≤ 2}, B = {(x, y) ∈ R2 | 0 < |x − 1| < 1},
C = {(x, y) ∈ R2 | 0 ≤ x ≤ y}, D = {(x, y, z) ∈ R3 /0 < x2 + y 2 + z 2 ≤ 1},
E = {(x, y, z) ∈ R /0 ≤ x + y + z ≤ 1},
3
F = {(x, y) ∈ R2 / x ≤ 0 et 0 ≤ y ≤ ex },
∆ = {(x, y) ∈ R2 | x 6∈ Q ou y 6∈ Q}.
.D
Solution. 1. Soit B(a, r) = {x ∈ Rn /kx − ak < r} une boule ouverte de Rn . Pour tout
x ∈ B(a, r), on a kx − ak < r, c’est-à-dire, r − kx − ak > 0. Choisissons alors 0 < ε <
r − kx − ak et montrons que B(x, ε) ⊂ B(a, r). En effet, pour tout y ∈ B(x, ε) on a
ky − xk < ε < r − kx − ak, alors ky − xk + kx − ak < r. D’où, ky − ak = ky − x + x − ak ≤
ky − xk + kx − ak < r. C’est-à-dire, y ∈ B(a, r) et par suite, la boule ouverte B(a, r) est
un ouvert de Rn .
Soit maintenant B 0 (a, r) = {x ∈ Rn /kx − ak ≤ r} une boule fermée de Rn . Pour tout
O
x ∈ [B 0 (a, r)]C "le complémentaire de la boule fermée", on a kx − ak > r, c’est-à-dire,
kx−ak−r > 0. Choisissons alors 0 < ε < kx−ak−r et montrons que B(x, ε) ⊂ [B 0 (a, r)]C .
En effet, pour tout y ∈ B(x, ε) on a ky − xk < ε < kx − ak − r, donc kx − ak − ky − xk > r.
Or, kx − ak = kx − y + y − ak ≤ kx − yk + ky − ak, alors, ky − ak ≥ kx − ak − ky − xk > r,
UI
c’est-à-dire, y ∈ [B 0 (a, r)]C et par suite, [B 0 (a, r)]C est un ouvert de Rn , d’où la boule
fermée B 0 (a, r) est un fermé de Rn .
2. ? A = {(x, y) ∈ R2 /|x| + |y| ≤ 2} = {(x, y) ∈ R2 /k(x, y)k1 ≤ 2} = Bk0 k1 ((0, 0), 2).
RI

La partie A est-elle ouverte ? Il existe un point (a, b) ∈ A (on peut choisir (a, b) = (2, 0))
tel que pour tout ε > 0, Bk k∞ ((2, 0), ε) * A. En effet, (2 + 2ε , 0) ∈ Bk k∞ ((2, 0), ε), mais

13
S.M. DOUIRI

(2 + 2ε , 0) ∈
/ A.
La partie A est-elle fermée ? A est la boule fermée de centre (0, 0) et de rayon 2 pour la norme
k k1 , donc c’est un fermé de R2 . Elle est aussi bornée, puisque k(x, y)k1 ≤ 2 pour tout (x, y) ∈ A.
Alors, A est un compact.
? B = {(x, y) ∈ R2 | 0 < |x − 1| < 1}
= {(x, y) ∈ R2 | −1 < x − 1 < 0 ou 0 < x − 1 < 1}
= {(x, y) ∈ R2 | 0 < x < 1 ou 1 < x < 2}
= {(x, y) ∈ R2 | x ∈]0, 1[∪]1, 2[}
S.
M
.D
B est un ouvert de R2 . En effet, pour tout (a, b) ∈ B, on a a ∈]0, 1[∪]1, 2[. Or ]0, 1[∪]1, 2[
est un ouvert de R, alors il existe ε > 0 tel que ]a − ε, a + ε[⊂]0, 1[∪]1, 2[. Montrons que
BR2 ,k k∞ ((a, b), ε) ⊂ B. Soit (x, y) ∈ BR2 ,k k∞ ((a, b), ε), alors k(x, y) − (a, b)k∞ < ε, c’est-à-dire,
k(x − a, y − b)k∞ < ε. Donc, |x − a| < ε, ce qui entraine que x ∈]a − ε, a + ε[, d’où x ∈]0, 1[∪]1, 2[
et par suite (x, y) ∈ B. Alors, pour tout (a, b) ∈ B, il existe ε > 0 tel que BR2 ,k k∞ ((a, b), ε) ⊂ B,
ce qui signifie que B est un ouvert de R2 . Maintenant, est-il un fermé de R2 ? Remarquons que
( n1 , 0)n∈N∗ est une suite dans B qui converge vers (0, 0) (à vérifier), mais la limite (0, 0) ∈ / B.
O
Alors cette partie n’est pas fermée dans R2 . Donc B n’est pas aussi compact.
? C = {(x, y) ∈ R2 | 0 ≤ x ≤ y}
UI
RI

La partie C n’est pas ouvert. En effet, le point (0, 0) ∈ C et pour tout ε > 0 on a
Bk k∞ ((0, 0), ε) * C, puisque (− 2ε , 0) ∈ Bk k∞ ((0, 0), ε) et (− 2ε , 0) ∈
/ C. D’où, la partie C
n’est pas un ouvert dans R2 .
Par contre, C est une partie fermée de R2 . En effet, soit (xn , yn )n∈N une suite dans C qui

14
S.M. DOUIRI

converge vers une limite (a, b). Donc lim xn = a et lim yn = b.


n→∞ n→∞
Comme (xn , yn ) ∈ C pour tout n ∈ N, alors

0 ≤ xn ≤ yn , ∀n ∈ N,

ce qui implique que


0 ≤ lim xn ≤ lim yn ,
n→∞ n→∞

c’est-à-dire,
0 ≤ a ≤ b.
Ce qui entraine que (a, b) ∈ C, et par suite la partie C est fermée dans R2 . Même si elle est
fermée, elle n’est pas compact car C est non bornée. En effet, pour tout M > 0 il existe
S.
(x, y) ∈ C tel que k(x, y)k∞ > M. Il suffit de choisir par exemple (x, y) = (M, M + 1).
? D = {(x, y, z) ∈ R3 /0 < x2 + y 2 + z 2 ≤ 1}
= {(x, y, z) ∈ R3 /0 < k(x, y, z)k22 ≤ 1}
= {(x, y, z) ∈ R3 /0 < k(x, y, z)k2 ≤ 1}
M
= Bk0 k2 ((0, 0, 0), 1) \ {(0, 0, 0)}
.D
O
UI

La partie D est la boule unité fermée privée de son centre. Elle n’est pas ouvert. Il
suffit de trouver un point de D tel que toute boule de centre ce point n’est pas inclue
RI
dans D. le point (0, 0, 1) ∈ D et pour tout ε > 0 on a BR3 ,k k2 ((0, 0, 1), ε) * D. En effet,
on a (0, 0, 1 + 2ε ) ∈ BR3 ,k k2 ((0, 0, 1), ε) mais (0, 0, 1 + 2ε ) ∈
/ D.
La partie D n’est pas aussi fermée. Il suffit de trouver une suite dans D qui converge vers
une limite, mais cette limite n’appartient pas à D. On considère la suite ( n1 , 0, 0)n∈N∗ ⊂ D,
elle converge vers le point (0, 0, 0), puisque
s
1 1 1
lim k( , 0, 0) − (0, 0, 0)k2 = n→∞
lim ( )2 = n→∞
lim = 0.
n→∞ n n n

Par contre le point (0, 0, 0) ∈


/ D. La non fermeture de D entraine qu’elle est non compact.

15
S.M. DOUIRI
S.
? E = {(x, y, z) ∈ R3 /0 ≤ x + y + z ≤ 1}.

La partie E est évidement fermé. Pour toute suite (xk , yk , zk )k∈N d’éléments de E qui
converge vers une limite (a, b, c), on a 0 ≤ xk + yk + zk ≤ 1. En passant à la limite, on
M
obtient 0 ≤ lim (xk + yk + zk ) ≤ 1, c’est-à-dire, 0 ≤ lim xk + lim yk + lim zk ≤ 1. D’où,
k→∞ k→∞ k→∞ k→∞
0 ≤ a + b + c ≤ 1, ce qui implique que (a, b, c) ∈ E. Malgré la fermeture de E, il n’est pas
compact puisqu’il n’est pas borné. En effet, pour tout M > 0, il existe (x, y, z) ∈ E tel
que k(x, y, z)k∞ > M. Il suffit de choisir (x, y, z) = (−M, M + 21 , 0).
La partie E n’est pas ouverte, en utilisant l’origine (0, 0, 0) ∈ E et pour tout ε > 0 il
.D
existe (− 2ε , 0, 0) ∈ B((0, 0, 0), ε) tel que (− 2ε , 0, 0) ∈
/ E. D’où B((0, 0, 0), ε) * E.
? F = {(x, y) ∈ R2 / x ≤ 0 et 0 ≤ y ≤ ex }.
O
UI
RI

Soit (Wn )n∈N une suite de F qui converge vers ` = (x, y) ∈ R2 , alors Wn = (xn , yn ) avec
xn ≤ 0 et 0 ≤ yn ≤ exn , lim(xn ) = x et lim(yn ) = y. Par passage à la limite aux deux
inégalités, on trouve que x ≤ 0 et 0 ≤ y ≤ ex , c’est-à-dire que ` = (x, y) ∈ F . On conclut
alors que F est un fermé de R2 . Est-il borné ? Pour tout M > 0, il existe (x, y) ∈ F tel
que k(x, yk)∞ > M. Il suffit de prendre (x, y) = (−M − 1, 0). Par conséquence, F est non
compact.

16
S.M. DOUIRI

F est aussi non ouvert d’après l’existence d’un point (a, b) ∈ F ((a, b) = (0, 0)) tel que
pour tout ε > 0 on a B((0, 0), ε) * F, puisque ( 2ε , 0) ∈ B((0, 0), ε) mais ( 2ε , 0) ∈
/ F.
? ∆ = {(x, y) ∈ R2 | x ∈/ Q ou y ∈/ Q}.
On a ∆ = {R2 où Γ = {(x, y) ∈ R2 | x ∈ Q et y ∈ Q}. On sait que Q et R\Q sont denses
Γ

dans R, c’est-à-dire, pour tous a < b dans R, il existe r ∈ Q et x ∈ R\Q tels que a < r < b
et a < x < b. Montrons que Γ n’est pas un ouvert de R2 . Prenons le point (0, 0) ∈ D, pour
tout ε > 0 il existe x ∈ R\Q tel que 0 < x < ε. Alors (x, 0) ∈ Bk k1 ((0, 0), ε), puisque
k(x, 0) − (0, 0)k1 < ε, mais on a (x, 0) ∈
/ Γ et par suite Bk k1 ((0, 0), ε) * Γ.
La partie Γ est aussi non fermée. En effet, d’après la densité√de Q dans R, il existe une
suite (rn )n∈N dans Q qui converge√ vers le nombre
√ irrationnel 2. Alors (rn , 0)n∈N est une
suite dans D qui converge vers ( 2, 0), mais ( 2, 0) ∈ / Γ. Ce qui entraine que Γ n’est pas
fermée dans R2 .
S.
Comme Γ n’est ni ouvert, ni fermé. Alors, il en est de même pour ∆, il n’est ni ouvert, ni
fermé. ∆ n’est pas aussi compact.
M
.D
O
UI
RI

17
Chapitre 2
Fonctions de plusieurs variables réelles
S.
Exercice 2.0.1. 1. On se restreint aux fonctions de 2 variables (sans perte de généralité),
montrer que
lim f (x, y) = l =⇒ lim (lim f (x, y)) = lim(lim f (x, y)) = l.
M
(x,y)→(a,b) x→a y→b y→b x→a

2. Étudier la réciproque en utilisant la fonction f (x, y) = xy


x2 +y 2
.

Solution. 1. Supposons que lim f (x, y) = l, c’est-à-dire,


(x,y)→(a,b)
pour tout ε > 0 il existe η > 0, pour tout (x, y) tel que k(x, y) − (a, b)k∞ < η on a
.D
ε
|f (x, y) − l| < . Alors, pour tout x ∈ R et tout y ∈ R tels que |x − a| < η et |y − b| < η,
2
ε
on a |f (x, y) − l| < . Ce qui entraine que pour tout x ∈ R tel que |x − a| < η on a
2
ε
lim |f (x, y) − l| = | lim f (x, y) − l| ≤ < ε. Par suite, lim (lim f (x, y)) = l. De même pour
y→b y→b 2 x→a y→b
lim(lim f (x, y)) = l.
y→b x→a
O
2. En utilisant la fonction f (x, y) = xy
x2 +y 2
au voisinage de l’origine (0, 0), pour
tout x ∈ R on a lim f (x, y) = 0 et pour tout y ∈ R on a lim f (x, y) = 0, alors lim (lim f (x, y)) =
y→0 x→0 x→0 y→0
lim(lim f (x, y)) = 0. Malgré ça, la fonction f (x, y) = xy
n’admet pas de limite au point (0, 0).
UI
y→b x→a x2 +y 2
La réciproque est en général fausse.

Exercice 2.0.2.
1. Déterminer le domaine de définition pour chacune des fonctions suivantes, puis dire en le
justifiant si elle admet ou non un prolongement continu sur R2 :
RI
2
sin (xy) (x2 + y 2 ) x+y
a) f (x, y) = ; b) f (x, y) = ; c) f (x, y) = 2 ;
|x| + |y| 2
x −y 2 x + y2
!
y |y| x (sin y − y)
d) f (x, y) = 2 exp − 2 ; e) f (x, y) = .
x x x2 + y 2

x2 y

si (x, y) 6= (0, 0) ;
2. Soit g la fonction définie sur R par g(x, y) =
2
x4 + y 2 Montrer la

 0 si (x, y) = (0, 0).
continuité de la restriction de g à toute droite passante par l’origine, mais la fonction g
n’est pas continue en (0, 0).

18
S.M. DOUIRI

Solution. 1. (a) Df = {(x, y) ∈ R2 /|x| + |y| =


6 0}.

= {(x, y) ∈ R2 /|x| = 6 0 ou |y| = 6 0}


= {(x, y) ∈ R /x 6= 0 ou y 6= 0}
2

= {(x, y) ∈ R2 /(x, y) 6= (0, 0)}


= R2 \{(0, 0)}.
La fonction (x, y) −→ sin(xy) est continue sur R2 comme composée de deux fonctions
1
continues. La fonction (x, y) −→ est continue sur R2 \{(0, 0)} (fraction rationnelle).
|x| + |y|
Alors f est bien définie et continue sur son domaine de définition Df . !
sin(xy) sin(xy) xy
lim f (x, y) = lim = lim × (∗)
(x, y) → (0, 0) (x, y) → (0, 0) |x| + |y| (x, y) → (0, 0) xy |x| + |y|
S.
(x, y) ∈ Df (x, y) 6= (0, 0) (x, y) 6= (0, 0)
sin(xy) xy
= lim × lim .
(x, y) → (0, 0) xy (x, y) → (0, 0) |x| + |y|
(x, y) 6= (0, 0) (x, y) 6= (0, 0)
sin(xy) sin(t)
D’une part, lim = 1, puisque lim = 1, et d’autre part,
M
(x, y) → (0, 0) xy t→0 t
(x, y) 6= (0, 0)

xy |xy|
=
|x| + |y| |x| + |y|
.D
|xy|

|x|
≤ |y|.
xy
Or lim |y| = 0, alors lim = 0.
(x,y)→(0,0) (x,y)→(0,0) |x| + |y|
Remarque : La limite (∗) nécessite que le point (x, y) n’appartient ni à l’axe (Ox), ni à l’axe
(Oy), c’est-à-dire, x 6= 0 et y 6= 0. Alors pour compléter le calcul de la limite de f en (0, 0), il
O
faut prouver que lim f (x, 0) = lim f (0, y) = 0.
x→0 y→0
Par conséquence, lim f (x, y) = 0. D’où, la fonction f est prolongeable par continuité sur
(x, y) → (0, 0)
(x, y) ∈ Df
UI
R2 à une fonction définie par

 sin(xy)
 si (x, y) 6= (0, 0) ;
f˜(x, y) = |x| + |y|


0 si (x, y) = (0, 0).
RI
(b) Df = {(x, y) ∈ R2 /x2 − y 2 6= 0}
= {(x, y) ∈ R2 /(x − y)(x + y) 6= 0}
= {(x, y) ∈ R2 /x − y 6= 0 et x + y 6= 0}
= {(x, y) ∈ R2 /x 6= y et x 6= −y}.
La fonction f est une fraction rationnelle, alors elle est bien définie et continue sur son domaine
de définition Df .
4 a4
F Si (a, +̄a) ∈/ Df tel que a 6= 0, alors lim f (x, y) = + = +∞. Donc f n’est pas
(x, y) → (a, +̄a) 0
x>y
prolongeable sur les droites y = +̄x privée de l’origine.

19
S.M. DOUIRI

F Si a = 0, on pose x = r cos(θ) et y = r sin(θ) avec r > 0 et θ ∈ [0, 2π[.


2 2
(x2 + y 2 ) (r2 )
lim f (x, y) = lim = lim (k ∈ Z)
(x, y) → (0, 0) (x, y) → (0, 0) x2 − y 2 r → 0, r2 (cos2 θ − sin2 θ)
(x, y) ∈ Df (x, y) ∈ Df π π
θ 6= + k
4 2
2
r
= lim = 0,
r → 0, cos2 θ − sin2 θ
π π
θ 6= + k
4 2

1 π π
puisque la quantité est bornée pour tout θ =
6 + k (k ∈ Z). Alors, f admet
cos2 θ − sin θ
2
4 2
S.
un prolongement continu sur Df ∪ {0} seulement (et pas sur R2 ), défini par
 2
(x2 + y 2 )


f˜(x, y) = 2 2
si (x, y) ∈ Df ;
 x −y

0 si (x, y) = (0, 0).
M
(c) Df = {(x, y) ∈ R2 /x2 + y 2 6= 0}
= {(x, y) ∈ R2 /x 6= 0 ou y 6= 0}
= {(x, y) ∈ R2 /(x, y) 6= (0, 0)}
= R2 \{(0, 0)}.
.D
La fonction f est une fraction rationnelle, alors elle est bien définie et continue sur son domaine
de définition Df .
Au point (0, 0), et suivant la droite y = 0, on a
x+y x 1
lim f (x, y) = lim = lim 2 = lim = +̄∞.
(x, y) → (0, 0) (x, y) → (0, 0) x + y
2 2 x→0 x x→0 x
y=0 y=0
O
Alors, f n’admet pas de prolongement continu au point (0, 0).
(d) Df = {(x, y) ∈ R2 /x2 6= 0}
= {(x, y) ∈ R2 /x 6= 0}
= R2 \{(0, y)/y ∈ R}.
UI
Les fonctions f1 : (x, y) 7−→ xy2 , f2 : (x, y) 7−→ − |y| x2
et f3 : t 7−→ exp(t) sont continues sur leurs
domaines de définitions, alors f = f1 × (f3 ◦ f2 ) est continue sur Df . Soit (0, b) ∈ / Df .
F Si b = 0 et selon les deux chemins y = x et ! y = x 2
, on a respectivement
!
y |y| x |x|  
1 1
lim f (x, y) = lim exp − = lim exp − = lim exp − = 0 et
(x, y) → (0, 0) x2 x2 x→0 x2 x2 x→0 x |x|
(x, y) → (0, 0)
RI
y=x y=x
! !
y |y| x2 |x2 |
lim f (x, y) = lim 2
exp − 2 = lim 2 exp − 2 = e−1 .
(x, y) → (0, 0) (x, y) → (0, 0) x x x→0 x x
y = x2 y = x2
Alors, la fonction f n’a pas de limite en (0, 0), donc elle n’admet pas de prolongement continu
sur (0, 0).
y |y|
F Si b 6= 0, on a lim 2
= +∞, alors en posant t = 2 , on aura
(x,y)→(0,b) x x
!
y |y|
lim |f (x, y)| = lim exp − = lim t e−t = 0.
(x,y)→(0,b) (x,y)→(0,b) x2 x2 t→+∞

20
S.M. DOUIRI

Ainsi, lim f (x, y) = 0 et donc la fonction f possède un prolongement par continuité sur
(x,y)→(0,b)
R2 \{(0, 0)} (et pas sur R2 ) défini par
 !
y  |y|
˜
exp − 2 si x 6= 0 ;
f (x, y) = x 2 x


0 si x = 0 et y 6= 0.
(e) Df = {(x, y) ∈ R2 /x2 + y 2 6= 0}
= {(x, y) ∈ R2 /x2 6= 0 ou y 2 6= 0}
= {(x, y) ∈ R2 /x 6= 0 ou y 6= 0}
= R2 \{(x, y) ∈ R2 /x = 0 et y = 0}.
= R2 \{(0, 0)}.
S.
1
Les fonctions f1 : (x, y) 7−→ x(sin y − y) et f2 : (x, y) 7−→ x2 +y 2 sont continues sur leurs

domaines de définitions, alors f = f1 × f2 est continue sur Df .


Au point (0, 0), en utilisant le développement limité d’ordre 2 de la fonction sin y au voisinage de
2 ε(y)
0, on a sin y = y+y 2 ε(y), avec lim ε(y) = 0. Ainsi, f (x, y) = xy
x2 +y 2
. Or, |xy| ≤ x2 +y 2 , ∀(x, y) ∈
y→0
M
R2 , alors |f (x, y)| ≤ |yε(y)| −−−−−−→ 0, ce qui entraine que lim f (x, y) = 0.
(x,y)→(0,0) (x,y)→(0,0)
Autrement, en utilisant les coordonnées polaires, on obtient
x(sin y − y) r cos θ(sin(r sin θ) − r sin θ)
lim f (x, y) = lim = lim
(x, y) → (0, 0) (x, y) → (0, 0) x2 + y 2 r → 0, r2
.D
(x, y) ∈ Df (x, y) ∈ Df θ ∈ [0, 2π[
! !
sin(r sin θ) sin(r sin θ)
= lim sin θ cos θ − 1 = 0, puisque −−→ 1 .
r → 0, r sin θ r sin θ r→0
θ ∈ [0, 2π[

Par conséquent, la fonction f admet un prolongement par continuité sur R2 défini par

 x(sin y − y)

si (x, y) 6= (0, 0) ;
f˜(x, y) = 
O
x2 + y 2
 0 sinon.
2. La fonction g est une fraction rationnelle sur R2 \{(0, 0)}, alors elle est continue en tout
point distinct de l’origine. Étudions la continuité de g en (0, 0) premièrement suivant les
UI
droites passantes par l’origine et deuxièmement sa continuité à l’origine. Choisissons une
droite y = α x, alors
x2 y α x3
lim g(x, y) = lim = lim
(x, y) → (0, 0) (x, y) → (0, 0) x4 + y 2 x→0 x4 + α2 x2
y = αx y = αx
RI
αx
= lim 2 = 0 = g(0, 0).
x→0 x + α2

Ce qui entraine que la restriction de g à toute droite passante par l’origine est continue en
(0, 0). Par contre la fonction g n’est pas continue au point (0, 0). En effet, selon le chemin de
la parabole y = x2 on a
x2 y x4
lim g(x, y) = lim = lim
(x, y) → (0, 0) (x, y) → (0, 0) x4 + y 2 x→0 x4 + x4
y = x2 y = x2
1
= 6= g(0, 0).
2
21
S.M. DOUIRI

Exercice 2.0.3.
1. Soit f : R → R une fonction de classe C 1 . On définit F : R2 → R par
( f (x)−f (y)
, si x 6= y;
F (x, y) = x−y
f (x),
0
sinon.

Démontrer que F est continue sur R2 .


2. Si f est seulement dérivable sur R, peut-on ( prouver le même résultat ?
x2 sin x1 , si x 6= 0;
(Ind. Utiliser la fonction définie par f (x) = )
0, sinon.

Solution. 1. Montrons d’abord que la fonction F est continue sur R2 \D où


S.
D = {(x, y) ∈ R2 / x = y}. Comme f est une fonction de classe C 1 sur R, alors f est
continue sur R. Or p1 et p2 , les fonctions de projection sur R2 , sont aussi continues sur R2 ,
alors les fonctions composées f ◦ p1 et f ◦ p2 sont continues sur R2 . Par conséquence, la fonction
f1 : (x, y) 7−→ f (x) − f (y) est continue sur R2 , car f1 = f ◦ p1 − f ◦ p2 . La fonction polynôme
f2 : (x, y) 7−→ x − y est continue sur R2 , alors la fonction F est continue sur R2 \D comme un
M
f1
quotient de deux fonctions continues f1 et f2 (F = sur R2 \D).
f2
Étudions maintenant, la continuité de F sur D. Soient (a, a) ∈ D et (x, y) dans un voisinage
de (a, a), c’est-à-dire, (x, y) ∈ B((a, a), ε). Si x = y alors lim F (x, y) = x→a
lim F (x, x) =
.D
(x, y) → (a, a)
x=y
lim f 0 (x) = f 0 (a) = F (a, a), d’après la continuité de f 0 . Si x 6= y, En appliquant le théorème
x→a
des accroissements finis à la fonction f , il existe Cx,y compris entre x et y tel que f (x) − f (y) =
f 0 (Cx,y ) .(x − y), c’est-à-dire,
F (x, y) = f 0 (Cx,y ) .
Quand (x, y) → (a, a), alors Cx,y → a et la continuité de f 0 entraine que
O
F (x, y) −−−−−−→ f 0 (a) = F (a, a). D’où la continuité de F sur D.
(x,y)→(a,a)
(
x2 sin x1 , si x 6= 0;
2. La fonction f définie par f (x) = est seulement
0, sinon
   
UI
dérivable mais n’est pas de classe C 1 sur R (à vérifier) avec f 0 (x) = 2x sin x1 − cos x1 , pour
tout x 6= 0. Montrons que la fonction F n’est pas
 continue  en (0, 0). En effet, les suites de R
2
 
1
données par (xk , xk ) = , 1
2kπ 2kπ
et (yk , 0) = π
1
+2 k π
, 0 convergent vers (0, 0), mais
2
   
1 1
lim F (xk , xk ) = lim f 0 = lim sin(2kπ) − cos(2kπ) = −1
n→+∞ k→+∞ 2kπ k→+∞ kπ
RI
   2  
f 1
− f (0) 1
sin π
+ 2kπ − 0 Ce qui
π
+2 k π π
+2kπ 2
et lim F (yk , 0) = lim = lim = 0.
2 2
1 1
k→+∞ k→+∞ −0 π
+2kπ
k→+∞ π
+2kπ
2 2
implique que F n’admet pas de limite en (0, 0).
Exercice 2.0.4. Soit f une fonction réelle d’une seule variable définie sur un intervalle ouvert.
Montrer que la différentiabilité de f est équivalente à sa dérivabilité.
Solution. Soient I un intervalle ouvert de R et f : I −→ R une fonction réelle d’une seule
f (x+h)−f (x)
variable définie sur I. Si f est dérivable en x ∈ I, alors on a lim
h→0 h
= f 0 (x) ∈ R.
x+h∈I

22
S.M. DOUIRI

f (x+h)−f (x)−f 0 (x)h


Donc, lim
h→0 |h|
= 0. Or l’application h 7−→ f 0 (x)h est une forme linéaire sur R, ce
x+h∈I
qui entraine que f est différentiable en x et on a
df (x) : R −→ R
h 7−→ df (x)(h) = f 0 (x)h.
f (x+h)−f (x)−df (x)(h)
Réciproquement, si f est différentiable en x ∈ I, alors on a lim
h→0 |h|
= 0. D’où
x+h∈I
f (x+h)−f (x)−df (x)(h)
lim
h→0 h
= 0. D’après la linéarité de df (x) on a
x+h∈I
f (x+h)−f (x)
df (x)(h) = hdf (x)(1), ∀h ∈ R, ce qui implique que lim
h→0 h
= df (x)(1). Par conséquent,
x+h∈I
S.
f est dérivable en x et on a f 0 (x) = df (x)(1).
Exercice 2.0.5. Déterminer la différentielle de la fonction f dans les cas suivants :
a) f : R2 → R avec f (x, y) = x + 2y + x2 y.
b) f : R2 → R2 avec f (x, y) = (ey , x2 ).
M
Solution. a) La fonction numérique f définie sur R2 par f (x, y) = x + 2y + x2 y est poly-
nômiale, alors elle est différentiable sur R2 . La différentielle de f est donnée par
df : R2 −→ (R)0
.D
(x, y) 7−→ df (x, y),
avec
df (x, y) : R2 −→ R
∂f ∂f
(h, k) 7−→ df (x, y)(h, k) =< ∇f (x, y)|(h, k) >= (x, y) h + (x, y) k,
∂x ∂y
O
Or ∂f
∂x
(x, y) = 1 + 2xy et ∂f
∂y
(x, y) = 2 + x2 , alors

df (x, y)(h, k) = (1 + 2xy) h + (2 + x2 ) k.


UI
b) La fonction vectorielle f définie sur R2 par f (x, y) = (f1 (x, y), f2 (x, y)),
où f1 (x, y) = ey et f2 (x, y) = x2 , est différentiable sur R2 , puisque ses fonctions composantes
f1 et f2 sont différentiables sur R2 . La différentielle de f est donnée par
df : R2 −→ L(R2 , R2 )
(x, y) 7−→ df (x, y),
RI
avec
df (x, y) : R2 −→ R2
!
h
(h, k) 7−→ df (x, y)(h, k) = Jf (x, y) .
k
! !
∂f1
(x, y) ∂f1
(x, y) 0 ey
Or Jf (x, y) = ∂x ∂y
= , alors
∂f2
∂x
(x, y) ∂f2
∂y
(x, y) 2x 0
! ! !
0 ey h k ey
df (x, y)(h, k) = = .
2x 0 k 2xh

23
S.M. DOUIRI

Exercice 2.0.6. Soit g la fonction définie par g(x, y) = xy ln(x2 + y 2 ).


1. Quel est le domaine de définition de la fonction g.
2. Montrer que g possède un prolongement par continuité sur R2 défini par
(
xy ln(x2 + y 2 ) si (x, y) 6= (0, 0) ;
f (x, y) =
0 sinon.

(Indication : Pour tout α > 0, on a lim+ tα ln t = 0.)


t→0
3. Etudier l’existence des dérivées partielles premières de f sur R2 .
4. Etudier la différentiabilité de f en chaque point de R2 et donner sa différentielle lorsqu’elle
existe.
S.
5. Déterminer le plus grand ouvert de R2 sur lequel f est de classe C 1 .
Solution. Soit g la fonction définie par g(x, y) = xy ln(x2 + y 2 ).
1. Soit Dg le domaine de définition de la fonction g. Alors
M
Dg = {(x, y) ∈ R2 /x2 + y 2 > 0}
= {(x, y) ∈ R2 /x2 + y 2 6= 0}
= {(x, y) ∈ R2 /x2 6= 0 ou y 2 6= 0}
= {(x, y) ∈ R2 /x 6= 0 ou y 6= 0}
.D
= R2 \{(x, y) ∈ R2 /x = 0 et y = 0}.
= R2 \{(0, 0)}.

2. Montrons que g possède un prolongement par continuité en (0, 0). On a

lim |g(x, y)| = lim xy ln(x2 + y 2 )


(x,y)→(0,0) (x,y)→(0,0)

= lim |r2 cos(θ) sin(θ) ln(r2 )| avec x = r cos(θ) et y = r sin(θ)


O
r→0
= lim |2r2 cos(θ) sin(θ) ln(r)|
r→0
≤ lim |2r2 ln(r)| = | lim 2r2 ln(r)|, car | cos(θ) sin(θ)| ≤ 1.
r→0 r→0
| {z }
UI
=0

Donc lim g(x, y) = 0. Ainsi g admet un prolongement par continuité sur R2 défini
(x,y)→(0,0)
par (
xy ln(x2 + y 2 ) si (x, y) 6= (0, 0) ;
f (x, y) =
0 sinon.
RI
3. Étudions l’existence des dérivées partielles premières de f sur R2 . D’abord sur R2 \{(0, 0)},
on a f = f1 × (f2 ◦ f3 ) avec f1 : (x, y) 7→ xy, f2 : t 7→ ln(t) et f3 : (x, y) 7→ x2 + y 2 . Comme
f1 et f3 sont des polynômes admettant des dérivées partielles sur R2 et f2 est dérivable
sur R∗+ , alors f admet des dérivées partielles premières par rapport aux variables x et y
et on a
∂f 2x
(x, y) = fy0 (x) = y ln(x2 + y 2 ) + xy · 2
∂x x + y2
2x2 y
= y ln(x2 + y 2 ) + 2 ,
x + y2

24
S.M. DOUIRI

et
∂f 2y
(x, y) = fx0 (y) = x ln(x2 + y 2 ) + xy · 2
∂y x + y2
2y 2 x
= x ln(x2 + y 2 ) + 2 ,
x + y2
où fx et fy sont les fonctions partielles de f .
Au point (x, y) = (0, 0), nous étudions les limites suivantes :

f (x, 0) − f (0, 0) 0
lim = lim = 0
x→0 x−0 x→0 x
f (0, y) − f (0, 0) 0
S.
lim = lim = 0.
y→0 y−0 y→0 y

∂f ∂f
Ce qui prouve que (0, 0) = (0, 0) = 0.
∂x ∂y
M
4. Étudions la différentiabilité de f en chaque point de R2 et donnons sa différentielle :
• Sur R2 \{(0, 0)} on a f = f1 ×(f2 ◦f3 ) avec f1 : (x, y) 7→ xy, f2 : t 7→ ln(t) et f3 : (x, y) 7→ x2 +y 2
sont des fonctions différentiables respectivement sur R2 , R+∗ et R2 ; et puisque f3 (R2 \{(0, 0)}) ⊂
R+∗ , alors f est différentiable sur R2 \{(0, 0)} et on a pour tout (x, y) ∈ R2 \{(0, 0)}
.D
! !
2 2x2 y
2 2 2 2y 2 x
df(x,y) (h1 , h2 ) = y ln(x + y ) + 2 .h1 + x ln(x + y ) + .h2 .
x + y2 x2 + y 2

• Pour la différentiabilité de f en (0, 0), nous étudions la limite suivante :

∂f ∂f
f (x, y) − f (0, 0) − (0, 0)x − (0, 0)y
∂x ∂y
O
lim ε(x, y) = lim
(x,y)→(0,0) (x,y)→(0,0) k(x, y)k2
xy ln(x2 + y 2 )
= lim √ 2
(x,y)→(0,0) x + y2
r2 cos(θ) sin(θ) ln(r2 )
UI
= lim √ avec x = r cos(θ) et y = r sin(θ).
r→0 r2
= lim 2r cos(θ) sin(θ) ln(r) = 0,
r→0

car la fonction θ 7→ cos(θ) sin(θ) est bornée et lim r ln(r) = 0. Alors f est différentiable en (0, 0)
r→0
RI
∂f ∂f
et on a df(0,0) (h1 , h2 ) = (0, 0)h1 + (0, 0)h2 = 0.
∂x ∂y
En résumant ce qui précède, la fonction f est différentiable sur R2 et on a pour tout h =
(h1 , h2 ) ∈ R2
 ! !
 2x2 y 2y 2 x
 y ln(x + y ) + 2
2 2
h1 + x ln(x 2
+ y 2
) + h2 si (x, y) 6= (0, 0) ;
df(x,y) (h) =  x + y2 x2 + y 2

0 sinon.

5. Déterminons maintenant le plus grand ouvert de R2 sur lequel f est de classe C 1 . D’après
la question 3 on a

25
S.M. DOUIRI

 2x2 y
∂f y ln(x2 + y 2 ) + 2

si (x, y) 6= (0, 0) ;
(x, y) =  x + y2
∂x  0 sinon.
2x2 y
Comme l’application (x, y) 7→ y ln(x2 + y 2 ) + est continue sur R2 \{(0, 0)}, alors il suffit
x2 + y 2
∂f
d’étudier la continuité de en (0, 0). Posons x = r cos(θ) et y = r sin(θ). Alors
∂x
∂f 2 2 2x2 y
lim (x, y) = lim y ln(x + y ) + 2
(x,y)→(0,0) ∂x (x,y)→(0,0) x + y2
2 2(r cos(θ))2 r sin(θ)
= lim r sin(θ) ln(r ) +
r2
S.
r→0
2
= lim 2r sin(θ) ln(r) + 2r cos (θ) sin(θ)
r→0
∂f
=0= (0, 0),
∂x
M
car les fonctions θ 7→ cos2 (θ) sin(θ) et θ 7→ sin(θ) sont bornées, lim r = 0 et lim r ln(r) = 0.
r→0 r→0
∂f ∂f
Donc est continue en (0, 0). On Déduit que est continue sur R . De la même façon nous
2
∂x ∂x
∂f
montrons que est continue sur R2 . Ce qui entraîne que f est de classe C 1 sur R2 .
.D
∂y
(  
1
x2 y 2 sin si x 6= 0 ;
Exercice 2.0.7. Soit f la fonction définie sur R2 par f (x, y) = x
0 sinon.
1. Calculer les dérivées partielles de f sur R2 .
2. Montrer que f n’est pas de classe C 1 sur R2 .
3. La fonction f est-elle différentiable sur R2 ?
O
Solution.
1. — Étudions d’abord l’existence de dérivées partielles premières sur R2 \D où D =
{(x, y) ∈ R2 : x = 0}.
UI
1
On a f = g × (sin ◦h) avec g(x, y) = x2 y 2 et h(x, y) = . Comme la fonction g est un
x
polynôme, h est une fraction rationnelle bien définie sur R2 \D et sin est une fonction usuelle,
alors f possède des dérivées partielles par rapport aux deux variables sur R2 \D, et on a

1 1 1
RI
∂f ∂f
(x, y) = 2xy 2 sin( ) − y 2 cos( ) et (x, y) = 2x2 y sin( ), ∀(x, y) ∈ R2 \D.
∂x x x ∂y x

— Sur D = {(x, y) ∈ R2 : x = 0} : Soit (0, y) ∈ D. On a


1
f (t, y) − f (0, y) t2 2
y sin( ) 1
lim = lim t = lim ty 2 sin( ) = 0,
t→0 t−0 t→0 t t→0 t
alors f possède au point (0, y) une dérivée partielle par rapport à la première variable et on a

∂f
(0, y) = 0.
∂x
26
S.M. DOUIRI

De même,
f (0, t) − f (0, y) 0−0
lim = lim = 0,
t→y t−y t→y t−y
alors f admet en (0, y) une dérivée partielle par rapport à la deuxième variable et on a
∂f
(0, y) = 0.
∂y
Autrement, en utilisant les fonctions partielles :
— Soit (x, y) ∈ R2 \D, c’est-à-dire, x 6= 0. Notons par f1 : R −→ R et
f2 : R −→ R les deux fonctions partielles de f en (x, y), qui sont définies par
(    
t2 y 2 sin 1
si t 6= 0 ; 2 2 1
f1 (t) = f (t, y) = t et f2 (t) = f (x, t) = x t sin .
S.
0 si t = 0, x
Comme f1 est dérivable sur R∗ et f2 est dérivable sur R, alors elles sont dérivables en x 6= 0 et y
respectivement. Ce qui implique que f admet au point (x, y) une dérivée partielle par rapport
aux deux variables et on a pour tout (x, y) ∈ R2 \D
M
∂f 1 1 ∂f 1
(x, y) = f10 (x) = 2xy 2 sin( ) − y 2 cos( ) et (x, y) = f20 (x) = 2x2 y sin( ).
∂x x x ∂y x
— Étudions maintenant l’existence de dérivées partielles premières sur D.
Soit (x, y) ∈ D, c’est-à-dire, x = 0. Notons toujours par f1 : R −→ R et f2 : R −→ R les deux
.D
fonctions partielles de f en (0, y), qui sont définies par
(  
1
t2 y 2 sin si t 6= 0 ;
f1 (t) = f (t, y) = t et f2 (t) = f (0, t) = 0.
0 si t = 0,
Comme f2 est dérivable sur R alors elle est dérivable en y, ce qui implique que f admet au point
∂f
(0, y) une dérivée partielle par rapport à la deuxième variable et on a (0, y) = f20 (y) = 0. Pour
∂y
O
1
f1 (t) − f1 (0) f (t, y) − f (0, y) t2 y 2 sin( )
la dérivabilité de f1 en x = 0, on a lim = lim = lim t =
t→0 t−0 t→0 t−0 t→0 t
1 1
lim ty 2 sin( ) = 0, puisque |ty 2 sin( )| ≤ |ty 2 | et lim |ty 2 | = 0. Alors f possède au point (0, y)
UI
t→0 t t t→0
∂f
une dérivée partielle par rapport à la première variable et on a (0, y) = f10 (0) = 0.
∂x
2. On a  
1 1 1
∂f 
2xy 2 sin( ) − y 2 cos( ) si x 6= 0 ; ∂f 
2x2 y sin( ) si x 6= 0 ;
(x, y) = x x et (x, y) = x
 
∂x 0 sinon. ∂y 0 sinon.
RI
∂f ∂f
Alors, et sont continues sur R2 \D, ce qui montre que f est de classe C 1 sur l’ouvert R2 \D.
∂x ∂y
Étudions maintenant la continuité des dérivées partielles sur D, pour cela prenons (0, y) ∈ D.
On a    
∂f 2 1 2 1
lim (x, t) = lim 2xt sin − t cos
(x,t)→(0,y) ∂x (x,t)→(0,y) x x
   
2 1 2 1
= lim 2xt sin − lim t cos
(x,t)→(0,y) x (x,t)→(0,y) x
 
1
= −y 2 lim cos
x→0 x

27
S.M. DOUIRI
 
1 ∂f
Or, lim cos n’existe pas, alors n’est pas continue en (0, y) pour tout y 6= 0. Ce qui
x→0 x ∂x
∂f
montre que n’est pas continue sur R2 , et par suite f n’est pas de classe C 1 sur R2 .
∂x
∂f ∂f
Remarque : Si y = 0 alors sera continue en (0, 0) et malgré que est aussi continue en
∂x ∂y
(0, 0), mais le plus grand ouvert sur lequel f est de classe C 1 est seulement R2 \D et n’est pas
R2 \D ∪ {(0, 0)}, car ce dernier n’est pas un ouvert.

3. La fonction f est-elle différentiable sur R2 ?

— On a déjà vérifié que la fonction f est de classe C 1 sur R2 \D, alors elle est différentiable
S.
sur cet ouvert.
— Reste à étudier la différentiabilité de f sur D.
Soit (0, y0 ) ∈ D, et étudions la limite en (0, 0) de l’application ε : R2 ! −→ R, définie par
∂f ∂f
M
f ((0, y0 ) + (x, y)) − f (0, y0 ) − x. (0, y0 ) + y. (0, y0 )
∂x ∂y
ε(x, y) = (en utilisant la norme k k∞ )
k(x, y)k !
∂f ∂f
f (x, y0 + y) − f (0, y0 ) − x. (0, y0 ) + y. (0, y0 )
∂x ∂y
= ,
.D
k(x, y)k∞
f (x, y0 + y)
=
k(x, y)k∞  
1
x (y0 + y) sin
2 2

ε(x, y) = x .
k(x, y)k∞
x2 (y0 + y)2
On a |ε(x, y)| ≤ . Or k(x, y)k∞ = sup(|x|, |y|) alors |x| ≤ k(x, y)k∞ , ce qui implique
O
k(x, y)k∞
x2 (y0 + y)2 x2 (y0 + y)2
que ≤ , c’est-à-dire, |ε(x, y)| ≤ |x|(y0 + y)2 . Comme lim |x|(y0 +
k(x, y)k∞ |x| (x,y)→(0,0)
y)2 = 0, alors lim ε(x, y) = 0.
(x,y)→(0,0)
UI
Par conséquence, la fonction f est différentiable à n’importe quel point (0, y0 ) de D, et par suite
f est différentiable sur R2 .
Exercice 2.0.8. (Fonctions homogènes)/Facultatif
Une fonction f : Rn −→ R est dite homogène de degré α si
f (t x1 , t x2 , . . . , t xn ) = tα f (x1 , x2 , . . . , xn ), ∀t > 0.
RI
1. Quelles sont les homogènes, en précisant le degré, parmi les fonctions :
Q 1
(a) f (x1 , . . . , xn ) = ( ni=1 xi ) n , (b) f (x, y, z) = 2x2 − 3yz,
x y
(c) f (x, y) = xx−y 2 +y 2 , (d) f (x, y, z) = u( ) où u est une fonction de R dans R.
z z
2. Dans la suite, on se restreint (sans perte de généralité) aux fonctions de 2 variables et
on suppose que f est de classe C 1 . Montrer que si f est homogène de degré α, alors ses
dérivées partielles sont homogènes de degré α − 1.
3. Pour (a, b) ∈ R2 fixé, Exprimer g 0 (t) en fonction des dérivées partielles de f, où g est la
fonction réelle d’une seule variable définie sur R∗+ par g(t) = f (ta, tb).

28
S.M. DOUIRI

4. Établir la relation d’Euler suivante :


∂f ∂f
f est homogène de degré α ⇐⇒ x +y = α f.
∂x ∂y
(Indication pour la réciproque : Prouver que g vérifier sur R∗+ l’équation différentielle
t g 0 (t) = α g(t).)
Solution. 1. Les quatre fonctions sont homogènes, de degrés respectifs 1, 2, -1 et 0.
2. Soit f : R2 −→ R une de classe C 1 . Si f est homogène de degré α, c’est-à-dire,
f (t x, t y) = tα f (x, y), ∀t > 0, alors les fonctions ut et vt définies sur R2 par ut (x, y) =
f (t x, t y) et vt (x, y) = tα f (x, y) sont égales et de classe C 1 pour chaque t > 0 fixé. Ce qui
∂ut ∂vt ∂ut ∂vt
implique que = et = . Par la règle de dérivation en chaîne, on obtient
S.
∂x ∂x ∂y ∂y
∂ut ∂f ∂ut ∂f ∂vt ∂f
(x, y) = t (t x, t y) et (x, y) = t (t x, t y). Pour vt on a (x, y) = tα (x, y)
∂x ∂x ∂y ∂y ∂x ∂x
∂vt α ∂f
et (x, y) = t (x, y). Finalement et par comparaison, on obtient
∂y ∂y
M
∂f ∂f ∂f ∂f
(t x, t y) = tα−1 (x, y) et (t x, t y) = tα−1 (x, y), ∀t > 0.
∂x ∂x ∂y ∂y
∂f ∂f
D’où les dérivées partielles et sont homogènes de degré α − 1.
.D
∂x ∂y
3. En posant g(t) = f (ta, tb), la fonction réelle d’une seule variable g est la composée de f
et la fonction vectorielle ϕ : R −→ R2 définie par ϕ(t) = (at, bt), qui est de classe C 1 , puisque
ses fonctions composantes de t −−1→ at et t −−2→ bt sont de classe C 1 . Comme f est de classe
ϕ ϕ

C 1 , alors g l’est aussi. En appliquant la règle de dérivation en chaîne on obtient


∂f ∂f
g 0 (t) = a (t a, t b) + b (t a, t b).
O
∂x ∂y
4. F Si la fonction f est homogène de degré α, alors g(t) = f (tx, ty) = tα f (x, y).
Ce qui implique que g 0 (t) = α tα−1 f (x, y). En comparant avec la dérivée obtenue en question
précédente, on en déduit que pour tout couple (x, y) et tout t > 0
UI
∂f ∂f
α tα−1 f (x, y) = x (t x, t y) + y (t x, t y).
∂x ∂y
Il suffit donc de prendre t = 1 pour obtenir la relation d’Euler demandée.
F Réciproquement, Supposons que la fonction f satisfait la relation d’Euler, c’est-à-dire,
RI
∂f ∂f
x (x, y) + y (x, y) = α f (x, y), ∀(x, y) ∈ R2 . D’après la question 3, on a pour tout
∂x ∂y
(x, y) ∈ R2 et tout t > 0
∂f ∂f
g 0 (t) = x (t x, t y) + y (t x, t y)
∂x ∂y !
1 ∂f ∂f
= tx (t x, t y) + ty (t x, t y)
t ∂x ∂y
1
= α f (tx, ty)
t
1
= α g(t).
t
29
S.M. DOUIRI
α
D’où, g est une solution de l’équation différentielle de premier ordre g 0 = g, ce qui est
t
équivalent à g(t) = C t , où C est une constante réelle quand peut la déterminer par C =
α

g(1) = f (x, y). Alors pour tout (x, y) ∈ R2 et tout t > 0 on a g(t) = tα g(1), ce qui donne
f (t x, t y) = tα f (x, y), c’est-à-dire, la fonction f est homogène de degré α.
S.
M
.D
O
UI
RI

30
Chapitre 3
Calculs des intégrales doubles et triples
S.
y4
Exercice 3.0.1. On définit la fonction f de R2 dans R par f (x, y) = x2 − 2xy + .
4
M
1. Montrer que f admet trois points critiques.
2. Étudier l’existence des extrema locaux.
3. Les extrema locaux obtenus sont-ils globaux ?
.D
Solution. La fonction f est clairement de classe C ∞ , ce qui légitime les calculs qui suivent.

∂f


 (x, y) = 2x − 2y (
y=x
1. ∂x ⇐⇒ √ √



∂f
(x, y) = −2x + y 3 x = 0 ou x = 2 ou x = − 2.
∂y
√ √ √ √
La fonction f possède alors trois points critiques (0, 0), ( 2, 2) et (− 2, − 2).
∂ 2f ∂ 2f ∂ 2f ∂ 2f
O
2. On a (x, y) = 2, (x, y) = 3y 2
et (x, y) = (x, y) = −2.
∂x2 ∂y 2 ∂x∂y ∂y∂x
!
2 −2
Alors la matrice Hessienne de f en (x, y) est Hf (x, y) = .
−2 3y 2
UI
!
2 −2
— Pour le point critique (0, 0) on a Hf (0, 0) = . Alors
−2 0
det(Hf (0, 0)) = −4 < 0 d’où f n’admet pas d’extremum local en (0, 0), mais seulement elle
possède un point selle.
!
RI
√ √ √ √ 2 −2
— Pour le point critique ( 2, 2) on a Hf ( 2, 2) = . Donc,
−2 6
√ √
det(Hf (1, 1) = 8 > 0, et puisque 2 > 0 alors f admet un minimum local en ( 2, 2).
√ √
— De même pour le point critique (− 2, − 2).
3. On a
y4 y4
f (x, y) = x2 − 2xy + = x2 − 2xy + y 2 − y 2 + +1−1
4 !2 4
y2
= (x − y)2 + − 1 − 1.
2

31
S.M. DOUIRI
√ √ √ √
Par positivité des carrés et le fait que f ( 2, 2) = f (− 2, − 2) = −1, on en déduit que pour
tout couple (x, y) ∈ R2
!2
√ √ y2
f (x, y) − f ( 2, 2) = f (x, y) + 1 = (x − y)2 + −1 ≥ 0,
2
et !2
√ √ 2 y2
f (x, y) − f (− 2, − 2) = f (x, y) + 1 = (x − y) + −1 ≥ 0.
2
√ √ √ √
Ce qui prouve que les minima f ( 2, 2) et f (− 2, − 2) sont globaux.
Exercice 3.0.2. (Facultatif )
Soit f la fonction définie sur R2 par f (x, y) = x2 (y 2 + ex ).
S.
1. Montrer que (a, b) est un point critique de f si, et seulement si,
(a, b) ∈ {0} × R ∪ {(−2, 0)}.
2. Soit (a, b) un point critique de f .
M
(a) Si (a, b) ∈ {0} × R, vérifier que f admet un minimum global en (a, b).
(b) Si (a, b) = (−2, 0), la fonction f possède-elle un extremum en (−2.0)?
Solution. Soit f la fonction définie sur R2 par f (x, y) = x2 (y 2 + ex ).
1. (a, b) est un point critique de f si, et seulement si,
.D

∂f


 (a, b) = 0 (
2a(b2 + ea ) + a2 ea = 0
∂x ⇔
 ∂f

 (a, b) = 0 a2 × 2b = 0.
∂y
C’équivalent à
( ( (
O
2a(b2 + ea ) + a2 ea = 0 a=0 b=0
⇔ ou
a = 0 ou b = 0 b∈R 2aea + a2 ea = 0.
(
b=0
⇔ (a, b) ∈ {0} × R ou ⇔ (a, b) ∈ {0} × R ou (a, b) = (−2, 0).
2a + a2 = 0.
UI
2. Soit (a, b) un point critique de f .
(a) Si (a, b) ∈ {0}×R,, alors f (a, b) = 0. Or f (x, y) = x2 (y 2 +ex ) ≥ 0 = f (a, b), ∀(x, y) ∈
R2 . Donc f admet un minimum global en (a, b).
(b) Si (a, b) = (−2, 0). On a
RI
∂ 2f
• (x, y) = (2x + 2)(y 2 + ex ) + (x2 + 2x)ex
∂x2
∂ 2f ∂ 2f ∂ 2f
• (x, y) = 2x 2
et (x, y) = (x, y) = 4xy. Alors
∂y 2 ∂x∂y ∂y∂x
!
(2x + 2)(y 2 + ex ) + (x2 + 2x)ex 4xy
Hf (x, y) = .
4xy 2x2

−2e−2 0
Ce qui implique que det Hf (−2, 0) = = −16e−2 < 0, donc la fonction f
0 8
ne possède pas d’extremum en (−2, 0), mais seulement un point selle.

32
S.M. DOUIRI

Exercice 3.0.3. (Facultatif )


Étudier l’existence des extrema aux fonctions f : R2 → R suivantes :
1. f (x, y) = 3xy − x3 − y 3 ;
2. f (x, y) = y(x2 + (ln y)2 ) ;
3. f (x, y) = sin x + y 2 − 2y + 1 ;
4. f (x, y) = x4 + y 4 − 4xy ;
5. f (x, y, z) = 1 + 2y − 3y 2 + 2xz − 3z 2 .

Solution. 1. Cherchons d’abord les points critiques de la fonction définie par f (x, y) =
3xy − x3 − y 3 . Comme f est un polynôme, alors f est de classe C ∞ (R2 ), donc f est
différentiable sur R2 , ainsi
S.

 ∂f



 (x, y) = 0 (
 ∂x 3y − 3x2 = 0
df (x, y) = 0L(R2 ,R) ⇔ ⇔
 3x − 3y 2 = 0
M

 ∂f


 (x, y) = 0
∂y
( ( (
y = x2 y = x2 x = 0 ou x = 1
⇔ ⇔ ⇔
x − x4 = 0 x(1 − x3 ) = 0 y = x2
.D
La fonction f possède alors deux points critiques (0, 0) et (1, 1).
f est de classe C 2 (R2 ), déterminons donc la matrice Hessienne Hf (x, y) pour tout (x, y) ∈ R2 ,
ainsi
∂ 2f ∂ 2f ∂ 2f ∂ 2f
(x, y) = −6x, (x, y) = −6y et (x, y) = (x, y) = 3.
∂x2 ∂y 2 ∂x∂y ∂y∂x
Alors on a !
−6x 3
O
Hf (x, y) = .
3 −6y
!
0 3
• Pour le point critique (0, 0) on a Hf (0, 0) = . Donc det(Hf (0, 0)) = −9 < 0, d’où f
3 0
UI
n’admet pas d’extremum local en (0, 0), mais seulement elle
! possède un point selle.
−6 3
• Pour le point critique (1, 1) on a Hf (1, 1) = . Donc, det(Hf (1, 1)) = 27 > 0 et
3 −6
puisque −6 < 0 alors f admet un maximum local en (1, 1). Ce maximum est-il global ? On a
f (1, 1) = 1 et en remarquant que f (−2, 0) = 8 on déduit que f (1, 1) n’est pas un maximum
global.
RI
2. La fonction f (x, y) = y(x2 + (ln y)2 ) est définie sur R × R∗+ . Elle est aussi de classe C ∞
sur R × R∗+ . Ainsi

 ∂f

 (x, y) = 0 


 ∂x 
 2xy = 0
df (x, y) = 0L(R2 ,R) ⇔ ⇔ 1


 ∂f 
 x2 + (ln(y))2 + y × 2 ln(y) × =0


 (x, y) = 0 y
∂y
( (
x=0 x=0
⇔ ⇔
(ln(y))2 + 2 ln(y) = 0 ln(y)(ln(y) + 2) = 0

33
S.M. DOUIRI
( (
x=0 x=0
⇔ ⇔
ln(y) = 0 ou ln(y) = −2 y = 1 ou y = e−2
La fonction f possède alors deux points critiques (0, 1) et (0, e−2 ).
Pour la matrice Hessienne Hf (x, y) on a
∂ 2f ∂ 2f ln(y) 2 ∂ 2f ∂ 2f
(x, y) = 2y, (x, y) = 2 + et (x, y) = (x, y) = 2x.
∂x2 ∂y 2 y y ∂x∂y ∂y∂x
Alors on a  
2y 2x

Hf (x, y) =  2(ln(y) + 1) 
.
2x
y
!
2 0
S.
• Pour le point critique (0, 1) on a Hf (0, 1) = . Donc det(Hf (0, 1)) = 4 > 0 et puisque
0 2
2 > 0 alors f admet un minimum local en (0, 1). Est-il global ? Pour tout (x, y) ∈ R × R∗+ on a

f (x, y) − f (0, 1) = y(x2 + (ln y)2 ) − 0 = y(x2 + (ln y)2 ) ≥ 0,


M
ce qui entraine que le minimum f (0, 1) = 0 est global.  
2e−2 0
• Pour le point critique (0, e−2 ) on a Hf (0, e−2 ) =  −2 . Donc, det(Hf (0, e−2 ) =
0
e−2
.D
−2
2e−2 × −2 = −4 < 0, d’où f n’admet pas un extremum local en (0, e−2 ), mais seulement elle
e
possède un point selle.
3. La fonction f (x, y) = sin x + y 2 − 2y + 1 est de classe C ∞ sur R2 .

 ∂f (x, y) = cos x = 0

 


 ∂x  x = π + kπ où k ∈ Z
df (x, y) = 0L(R2 ,R) ⇔  ⇔ 2
 y=1
O
 ∂f


 (x, y) = 2y − 2 = 0
∂y
La fonction f possède alors une infinité de points critiques définie par
π
PC f = {( + kπ, 1) ∈ R2 /k ∈ Z}.
UI
2
Pour la matrice Hessienne Hf (x, y) on a
∂ 2f ∂ 2f ∂ 2f ∂ 2f
(x, y) = − sin x, (x, y) = 2 et (x, y) = (x, y) = 0.
∂x2 ∂y 2 ∂x∂y ∂y∂x
RI
Alors on a
!
− sin x 0 − sin x 0
Hf (x, y) = et det(Hf (x, y)) = = −2 sin x.
0 2 0 2
π π
— Pour les points critiques ( +(2k +1)π, 1), on a det(Hf ( + (2k + 1)π, 1)) = 2 et puisque
2 2
π π
− sin( + (2k + 1)π) = 1 > 0 alors f admet un minimum local en ( + (2k + 1)π, 1).
2 2
π
Est-il global ? On a f (x, y) ≥ −1 + (y − 1) ≥ −1 = f ( + (2k + 1)π, 1), alors le minimum
2
2
π
en ( + (2k + 1)π, 1) est global pour tout k ∈ Z.
2
34
S.M. DOUIRI
π π
— Pour les points critiques ( +2kπ, 1), on a det(Hf ( + 2kπ, 1)) = −2 < 0, d’où f n’admet
2 2
π
pas des extrema locaux aux points critiques ( + 2kπ, 1), ∀k ∈ Z, mais seulement elle
2
possède des points selles.
4. La fonction f (x, y) = x4 + y 4 − 4xy est de classe C ∞ sur R2 .

 ∂f



 (x, y) = 4x3 − 4y = 0 (
 ∂x x3 − y = 0
df (x, y) = 0L(R2 ,R) ⇔ ⇔

 y3 − x = 0
 ∂f (x, y) = 4y 3 − 4x = 0



∂y
( ( (
x3 = y x3 = y x3 = y
⇔ ⇔ ⇔
x9 − x = 0 x(x8 − 1) = 0 x = 0 ou x = ±1
S.
La fonction f possède alors trois points critiques (0, 0), (1, 1) et (−1, −1).

Pour la matrice Hessienne Hf (x, y) on a


M
∂ 2f 2 ∂ 2f 2 ∂ 2f ∂ 2f
(x, y) = 12x , (x, y) = 12y et (x, y) = (x, y) = −4.
∂x2 ∂y 2 ∂x∂y ∂y∂x
Alors on a !
12x2 −4
Hf (x, y) = .
.D
−4 12y 2
!
0 −4
• Pour le point critique (0, 0) on a Hf (0, 0) = . Donc det(Hf (0, 0)) = −16 < 0 d’où
−4 0
f n’admet pas un extremum local en (0, 0), mais seulement! elle possède un point selle.
12 −4
• Pour le point critique (1, 1) on a Hf (1, 1) = . Donc, det(Hf (1, 1) = 144 − 16 =
−4 12
128 > 0, et puisque 12 > 0 alors f admet un minimum local en (1, 1). Ce minimum est-il
O
global ? Pour tout (x, y) ∈ R2 on a
f (x, y) − f (1, 1) = x4 + y 4 − 4xy + 2 = x4 − 2x2 y 2 + y 4 + 2x2 y 2 − 4xy + 2
2
= (x2 − y 2 ) + 2 (x2 y 2 − 2xy + 1)
UI
2
= (x2 − y 2 ) + 2 (xy − 1)2
≥ 0,
ce qui entraine que le minimum f (1, 1) = −2 est global.

• Pour le point critique (−1, −1) on refait la même méthode que le point critique (1, 1), ou il
RI
suffit de remarquer que f (−(x, y)) = f (−x, −y) = f (x, y) pour tout (x, y) ∈ R2 , c’est-à-dire,
le graphe de f est symétrique par rapport à l’axe (Oz). D’où f admet aussi en (−1, −1) un
minimum global.
5. La fonction f (x, y, z) = 1 + 2y − 3y 2 + 2xz − 3z 2 est de classe C ∞ sur R3 .
On a 
∂f



 (x, y, z) = 0

 ∂x
 ∂f 1
df (x, y, z) = 0L(R3 ,R) ⇔ (x, y, z) = 0 ⇔ (x, y, z) = (0, , 0)

 ∂y
 3

 ∂f

 (x, y, z) = 0
∂z
35
S.M. DOUIRI

La matrice Hessienne de f au point critique (0, 31 , 0) est


 
0 0 2
Hf (x, y) = 
 0 −6 0 
.
2 0 −6

On cherche les valeurs propres de la matrice hessienne en A = (0, 13 , 0)

−λ 0 2 h i
0 −6 − λ 0 = (6 + λ) 4 − 6λ − λ2
2 0 −6 − λ
√ √
S.
Les valeurs propres de la hessienne sont −6, −3 + 13, −3 − 13. Deux sont négatives et l’autre
est positive. La condition suffisante n’est pas satisfaite. Utilisons alors une autre méthode.
Essayons de montrer que f ((0, 31 , 0) + (x, y, z)) − f (0, 13 , 0) garde le même signe ou change de
signe lorsque (x, y, z) tend vers (0, 0, 0). On a
M
1 1 −8
f ((0, , 0) + (x, y, z)) − f (0, , 0) = − 4y − 3y 2 + 2xz − 3z 2
3 3 3
En vérifiant que
.D
!
2 2 2xz − 4y − 3y 2 − 3z 2
− 4y − 3y + 2xz − 3z = k(x, y, z)k∞
k(x, y, z)k∞
= k(x, y, z)k∞ ε(x, y, z)

avec lim ε(x, y, z) = 0, on conclut que f possède un maximum local au point (0, 31 , 0).
(x,y,z)→(0,0,0)
O
Exercice 3.0.4. 1. Montrer que la condition ex+z = y −z définit z comme fonction de (x, y)
au voisinage de (1, 0, −1).
2. Quelle est la classe de cette fonction ? Calculer ses dérivées partielles.
UI
Solution.
1. En considérant la fonction f définie sur R3 par f (x, y, z) = ex+z − y + z,
on a la condition ex+z = y − z est équivalente à f (x, y, z) = 0. Comme f (1, 0, −1) = 0 et
∂f ∂f
(x, y, z) = ex+z + 1, c’est-à-dire, (1, 0, −1) = e0 + 1 = 2 6= 0, alors d’après le Théorème
RI
∂z ∂z
des fonctions implicites, il existe un voisinage U de (1, 0) dans R2 , un voisinage I de −1 dans
R et une fonction implicite ϕ : V −→ I vérifiant
i) V × I ⊂ R3 et ϕ(1, 0) = −1;
ii) f (x, y, z) = 0 ⇐⇒ z = ϕ(x, y), pour tout (x, y) ∈ V. C’est-à-dire, ex+z = y − z ⇐⇒ z =
ϕ(x, y), pour tout (x, y) ∈ V.
2. Toujours d’après le Théorème des fonctions implicites et comme la fonction
f définie par f (x, y, z) = ex+z − y + z, est de classe C ∞ sur R3 , alors la fonction implicite ϕ
est aussi de classe C ∞ sur V et on peut calculer ses dérivées partielles premières en utilisant

36
S.M. DOUIRI
!
∂f ∂f
(x, y, ϕ(x, y)), (x, y, ϕ(x, y))
∂x ∂y
l’expression ∇ϕ(x, y) = − , c’est-à-dire,
∂f
(x, y, ϕ(x, y))
∂z

∂ϕ ∂f
(x, y, ϕ(x, y)) ∂ϕ (x, y, ϕ(x, y))
∂f
(x, y) = − ∂x et (x, y) = − ∂y
∂x ∂f ∂y ∂f
(x, y, ϕ(x, y)) (x, y, ϕ(x, y))
∂z ∂z
ex+ϕ(x,y) −1
= − x+ϕ(x,y) = − x+ϕ(x,y)
e +1 e +1
Exercice 3.0.5. Soit ∆ une partie de R2 symétrique par rapport à l’axe (Oy) (resp. (Ox)),
c’est-à-dire, (x, y) ∈ ∆ ⇔ (−x, y) ∈ ∆ (resp. (x, y) ∈ ∆ ⇔ (x, −y) ∈ ∆). Soit f : R2 → R
S.
une fonction numérique vérifiant f (−x, y) = −f (x, y) (resp. f (x, −y) = −f (x, y)) pour tout
(x, y) ∈ ∆.
ZZ
1. Montrer que I = f (x, y) dxdy = 0.
M

ZZ
2. Application : Calculer l’intégrale double I = xy ln(x2 + y 2 ) dxdy

où ∆ = {(x, y) ∈ R2 / y > 0 et x2 + y 2 ≤ 1}.

Solution. 1. En effectuant un changement de variables via l’application injective de classe


.D
C sur R2 définie par ϕ(u, v) = (−u, v) = (x, y). On a ∆ = ϕ(∆) d’après l’équivalence
1

(u, v) ∈ ∆ ⇔ (−u, v) ∈ ∆ et det(Jϕ (u, v)) = −1. Alors,


ZZ ZZ
I= f (x, y) dxdy = f (x, y) dxdy
ZZ∆ ϕ(∆) ZZ
= f ◦ ϕ(u, v) |det(Jϕ (u, v))| dudv = f (−u, v) | − 1|dudv
ZZ∆ ZZ ∆
= −f (u, v) dudv = − f (u, v) dudv = −I.
O
∆ ∆

Ce qui entraine que I = 0.


2. Application : On a (x, y) ∈ ∆ ⇔ (x, y) ∈ R2 tel que y > 0 et x2 + y 2 ≤ 1 ⇔ (x, y) ∈
UI
R2 tel que y > 0 et (−x)2 + y 2 ≤ 1 ⇔ (−x, y) ∈ ∆.
Pour la fonction f (x, y) = xy ln(x2 +y 2 ), on a f (−x, y) = (−x)y ln((−x)2 +y 2 ) = −f (x, y).
Alors, par le changement de variables via l’application injective de classe C 1 sur R2 définie
par ϕ(u, v) = (−u, v) = (x, y), on a
ZZ ZZ
I= f (x, y) dxdy = f (x, y) dxdy
RI
ZZ∆ ϕ(∆) ZZ
= f ◦ ϕ(u, v) |det(Jϕ (u, v))| dudv = f (−u, v) | − 1|dudv
ZZ∆ ZZ ∆
= −f (u, v) dudv = − f (u, v) dudv = −I.
∆ ∆

D’où I = 0.

Exercice 3.0.6. Calculer l’aire ou le volume des ensembles suivants.


x2 y2
1. D1 = {(x, y) ∈ R2 : 2 + 2 ≤ 1}. Qu’obtient-on dans le cas particulier où D1 est le
a b
disque unité de R2 ?

37
S.M. DOUIRI

2. D2 = {(x, y, z) ∈ (R+ )3 : x + y + z ≤ 1}.


3. D3 = {(x, y, z) ∈ R3 : x2 + y 2 ≤ R et 0 ≤ z ≤ h}.

Solution. 1. On a
x2 y 2
D = {(x, y) ∈ R2 | + ≤ 1}
a22 b22
x x y2
= {(x, y) ∈ R2 | 2 ≤ 2 + 2 ≤ 1}
a a b s s
x2 x2
= {(x, y) ∈ R | −a ≤ x ≤ a, et − b 1 − 2 ≤ y ≤ b 1 − 2 }.
2
a a
Alors
q  q 
S.
ZZ Z a Z b 1− x2 Z a Z b 1− x2
2  2
Aire(D) = 1 dxdy = q a
2
dxdy =  q a
2
dy 
 dx
D −a −b 1− x2 −a −b 1− x2
a a
s s
Z a Z a Z 0√
x2 x2
= 2b 1 − dx = 2b dx = 2b 1−
1 − cos2 θ (−a sin θ)dθ
a2 2
M
−a
Z π
−a
Z
a π
" #π
π 1 − cos 2θ 1 sin 2θ
2
= 2ab sin θ dθ = 2ab dθ = 2ab θ − = abπ.
0 0 2 2 4 0

D’où
.D
Aire(D) = abπ.

2. On a
D= {(x, y, z) ∈ (R+ )3 | x + y + z ≤ 1}
= {(x, y, z) ∈ R3 | x ≥ 0, y ≥ 0, z ≥ 0 et x + y + z ≤ 1}
= {(x, y, z) ∈ R3 | x ≥ 0, y ≥ 0, z ≥ 0 et x ≤ x + y ≤ x + y + z ≤ 1}
= {(x, y, z) ∈ R3 | 0 ≤ x ≤ 1, 0 ≤ y ≤ 1 − x et 0 ≤ z ≤ 1 − x − y}
Alors, d’après le Théorème de Fubini on obtient
O
ZZ Z 1 Z 1−x Z 1−x−y Z 1 Z 1−x Z 1−x−y  
V(D) = 1 dxdydz = dxdydz = dz dy dx
0
Z 1DZ 1−x   0  0 Z 1 Z 1−x 0 0 0
1−x−y
= [z]0 dy dx = (1 − x − y) dy dx
UI
0 " 0 #1−x 0 0 !
Z 1 Z 1
y 2
(1 − x)2
= y − xy − dx = 1 − x − x(1 − x) − dx
0 2 0 0 2
" #1
x x2 x3 1
= − + = .
2 2 6 0 6
RI
D’où
1
V(D) = .
6

3. En utilisant le changement de variables en coordonnées cylindriques, via l’application in-


jective de classe C 1 sur R∗+ ×[0, 2π[×R définie par ϕ(r, θ, z) = (r cos θ, r sin θ, z) = (x, y, z),
on a D = {(x, y, z) ∈ R3 | x2 + y 2 ≤ R et 0 ≤ z ≤ h} = ϕ(D0 ) où

D0 = {(r, θ, z) ∈ R∗+ × [0, 2π[×R | r2 ≤ R et


√ 0 ≤ z ≤ h}
= {(r, θ, z) ∈ R+ × [0, 2π[×R | 0 < r ≤ R, 0 ≤ θ < 2π et 0 ≤ z ≤ h}.

38
S.M. DOUIRI

Alors
ZZZ ZZZ ZZZ
V(D) = 1 dxdydz = dxdydz = |det(Jϕ (r, θ, z))| drdθdz
D 0) D0 √
ZZZ Z √ ϕ(D
R Z 2π Z h Z R Z h Z 2π
= r drdθdz = r drdθdz = r dr dz dθ
D0 0 0 0 0 0 0
= πhR.
D’où
V(D) = πhR.

Exercice 3.0.7. (Facultatif )


Calculer les intégrales suivantes :
ZZ q
S.
1. I = sin x2 + y 2 dxdy avec D = {(x, y) ∈ R2 : π 2 < x2 + y 2 ≤ 4 π 2 }.
ZZD q
2. J = x2 + y 2 dxdy avec D = {(x, y) ∈ R2 : x2 + y 2 − y ≤ 0}.
D
ZZZ
1
3. K = dxdydz avec D = {(x, y, z) ∈ (R+ )3 : x + y + z ≤ 1}.
M
D (1 + x + y + z)2
ZZZ
2 +y 2
4. L = zx dxdydz avec D = {(x, y, z) ∈ R3 : x2 + y 2 ≤ 1 et 0 ≤ z ≤ 1}.
D
ZZZ q
5. M = (1 + x2 + y 2 + z 2 ) dxdydz avec D = {(x, y, z) ∈ R3 : y ≥ 0 et x2 + y 2 + z 2 ≤
.D
D
R2 }, (R > 0).
Solution. 1. En utilisant le changement de variables en coordonnées polaires, via l’appli-
cation injective de classe C 1 sur R∗+ × [0, 2π[ définie par ϕ(r, θ) = (r cos θ, r sin θ) = (x, y),
on a D = {(x, y) ∈ R2 | π 2 < x2 + y 2 ≤ 4 π 2 } = ϕ(D0 ) où

D0 = {(r, θ) ∈ R∗+ × [0, 2π[ | π 2 < r2 ≤ 4 π 2 }


= {(r, θ) ∈ R∗+ × [0, 2π[ | π < r ≤ 2 π et 0 ≤ θ < 2π}.
O
Alors, d’après le Théorème de Fubini on obtient
ZZ q ZZ q
I= sin( x2 + y 2 ) dxdy = sin( x2 + y 2 ) dxdy
UI
D ϕ(D0 )
ZZ ZZ √
= sin(r). |det(Jϕ (r, θ))| drdθ = sin r2 r drdθ
D0 D0
Z 2π Z 2π Z 2π Z 2π
= r sin r drdθ = r sin r dr dθ
π 0 π 0
 Z 2π 
= 2π [−r cos r]2π
π + cos r dr = 2π(−2π cos(2π) + π cos(π) + sin(2π) − sin(π))
RI
π

D’où
I = −6π 2 .
2. Par le changement de variables en coordonnées polaires, via l’application injective de
classe C 1 sur R∗+ × [0, 2π[ définie par ϕ(r, θ) = (r cos θ, r sin θ) = (x, y), on a
D = {(x, y) ∈ R2 : x2 + y 2 − y ≤ 0} = ϕ(D0 ) où

D0 = {(r, θ) ∈ R∗+ × [0, 2π[ | r2 − r sin θ ≤ 0}


= {(r, θ) ∈ R∗+ × [0, 2π[ | 0 < r ≤ sin θ et 0 < sin θ}
= {(r, θ) ∈ R∗+ × [0, 2π[ | 0 < r ≤ sin θ et 0 < θ < π}.

39
S.M. DOUIRI

Alors, d’après le Théorème de Fubini on obtient


ZZ q ZZ q ZZ √
J= x2 + y2 dxdy = x2 + y2 dxdy = r2 . |det(Jϕ (r, θ))| drdθ
D ϕ(D0 ) 0
! Z π" #D
ZZ Z π Z sin θ 3 sin θ
2 1Z π 3 r
= r × r drdθ = r dr dθ =
sin θ dθ dθ =
D0 0 0 0 3 0 " 3 0

1Z π 2 1Z π 2 1 cos3 θ
= sin θ(1 − cos θ)dθ = sin θ − sin θ cos θ dθ = − cos θ +
3 0 
3 0 3 3 0
1 −1 1 4
= 1+ +1− = .
3 3 3 9

D’où
4
S.
J= .
9

3. On a
D= {(x, y, z) ∈ (R+ )3 | x + y + z ≤ 1}
M
= {(x, y, z) ∈ R3 | x ≥ 0, y ≥ 0, z ≥ 0 et x + y + z ≤ 1}
= {(x, y, z) ∈ R3 | x ≥ 0, y ≥ 0, z ≥ 0 et x ≤ x + y ≤ x + y + z ≤ 1}
= {(x, y, z) ∈ R3 | 0 ≤ x ≤ 1, 0 ≤ y ≤ 1 − x et 0 ≤ z ≤ 1 − x − y}
Alors, d’après le Théorème de Fubini on obtient
.D
ZZZ Z 1 Z 1−x Z 1−x−y
1 1
K= dxdydz = dxdydz
D (1 + x + y + z)2 0 0 0 (1 + x + y + z)2
Z 1 Z 1−x Z 1−x−y ! !
1
= dz dy dx
0 0 0 (1 + x + y + z)2
  #1−x−y  
Z 1 Z 1−x "
−1
=    dy  dx
0 0 1+x+y+z
O
0
Z 1 Z 1−x ! !
−1 1
= + dy dx
0 0 2 1+x+y
Z 1 1−x
−1
= y + ln |1 + x + y|
UI
dx
0 2 0
Z 1 
1
= (x − 1) + ln(2) − ln(1 + x) dx
0 2
" #1
1 x2 1 1
= ( − x) + ln(2)x − ((1 + x) ln(1 + x) − x) = ( − 1) + ln(2) − (2 ln(2) − 1).
2 2 0
2 2
RI
D’où
3
K= − ln(2).
4

4. En utilisant le changement de variables en coordonnées cylindriques, via l’application in-


jective de classe C 1 sur R∗+ ×[0, 2π[×R définie par ϕ(r, θ, z) = (r cos θ, r sin θ, z) = (x, y, z),
on a D = {(x, y, z) ∈ R3 | x2 + y 2 ≤ 1 et 0 ≤ z ≤ 1} = ϕ(D0 ) où

D0 = {(r, θ, z) ∈ R∗+ × [0, 2π[×R | r2 ≤ 1 et 0 ≤ z ≤ 1}


= {(r, θ, z) ∈ R∗+ × [0, 2π[×R | 0 < r ≤ 1, 0 ≤ θ < 2π et 0 ≤ z ≤ 1}.

40
S.M. DOUIRI

Alors
ZZZ ZZZ ZZZ
x2 +y 2 x2 +y 2 2
L= z dxdydz = z dxdydz = z r . |det(Jϕ (r, θ, z))| drdθdz
D ϕ(D0 ) D0
ZZZ Z 1 Z 2π Z 1 Z 1Z 1 Z 2π
r2 r2 r2
= z .rdrdθdz = z .rdrdθdz = z .rdrdz dθ
D0 0 0 0 0 0 0
Z 1 Z 1  Z 1 Z 1 " 2 #1
z r +1
r2 1
= 2π r z dz dr = 2π r 2 dr = 2π r. 2 dr
0 0 0 r +1 0 0 r +1
Z 1 Z 1
r 2r h
2
i1
= 2π dr = π dr = π ln |r + 1| = π ln(2)
0 r2 + 1 0 r2 + 1 0

D’où
S.
L = π ln(2).

5. En utilisant le changement de variables en coordonnées sphériques, via l’application in-


jective de classe C 1 sur R∗+ × [0, 2π[×[0, π] définie par
M
ϕ : R∗+ × [0, 2π[×[0, π] −→ R3
(r, θ, φ) 7−→ ϕ(r, θ, φ) = (r cos θ sin φ, r sin θ sin φ, r cos φ) = (x, y, z),

on a D = {(x, y, z) ∈ R3 | y ≥ 0 et x2 + y 2 + z 2 ≤ R2 } = ϕ(D0 ) où
.D
D0 = {(r, θ, φ) ∈ R∗+ × [0, 2π[×[0, π] | r sin θ sin φ ≥ 0 et r2 ≤ R2 }
= {(r, θ, φ) ∈ R∗+ × [0, 2π[×[0, π] | sin θ ≥ 0 et 0 < r ≤ R}"car sin φ ≥ 0"
= {(r, θ, φ) ∈ R∗+ × [0, 2π[×[0, π] | 0 < r ≤ R, 0 ≤ θ ≤ π et 0 ≤ φ ≤ π}.
Alors, d’après le Théorème de Fubini on a
ZZZ q ZZZ q
M= (1 + x2 + y2 + z 2 ) dxdydz = (1 + x2 + y 2 + z 2 ) dxdydz
0)
ZZZD ϕ(D
ZZZ
O
= (1 + r) |det(Jϕ (r, θ, φ))| drdθdφ = (1 + r).r2 sin φ drdθdφ
Z R Z0
D
π Z π Z R D0 Z π Z π
= (r2 + r3 ) sin φ drdθdφ = (r2 + r ) dr
3
dθ sin φ dφ
"0 0 0# 0 ! 0 0
r3 r 4 R
R3 R4
UI
= + ×π× [− cos φ]π0 = 4π + .
3 4 0
3 4

D’où !
R3 R4
M = 4π + .
3 4
RI

41
Chapitre 4
Correction de certains examens
S.
4.1 Examen d’Analyse III (02 Mars 2021)
Exercice 4.1.1. [6 points]
M
Soient a ∈ Rn , f : Rn −→ R et g : Rn −→ Rp deux fonctions de plusieurs variables et A un
ensemble de Rn . Dire si les affirmations suivantes sont vraies V ou fausses F .
1. V Si A est ouvert, alors son complémentaire AC est non ouvert.
2. V Si (xk )k est une suite de A qui converge vers x, alors x ∈ A.
.D
3. V Si f ∈ C 1 (Rn ), alors f est différentiable en tout point de Rn .
4. V Si toutes les dérivées partielles premières existent en a, alors f est différentiable en a.
5. V Si g est différentiable en a, alors la Jacobienne Jg (a) existe.
6. V Si la Jacobienne Jg (a) existe, alors g est continue en a.
Exercice 4.1.2. [14 points] Justifier toutes les réponses.
O

 (x − 1)y 2
 , si (x, y) 6= (1, 0);
Soit f la fonction définie par f (x, y) =  (x − 1)2 + y 2

0, si (x, y) = (1, 0).
1. Quel est le domaine de définition de f.
UI
2. Étude de la fonction f sur R2 \{(1, 0)} :
2.1. Vérifier que f est continue sur R2 \{(1, 0)}.
2.2. Calculer les dérivées partielles premières de f sur R2 \{(1, 0)}.
2.3. Déduire que f est de classe C 1 sur R2 \{(1, 0)}.
RI
2.4. La fonction f est-elle différentiable sur R2 \{(1, 0)}?
3. Étude de la fonction f en (1, 0) :
3.1. Étudier la continuité de f en (1, 0), (Indication : Poser t = x − 1).
3.2. Étudier l’existence des dérivées partielles premières de f en (1, 0).
3.3. Montrer que f n’est pas différentiable en (1, 0).
3.4. Quel est le plus grand ouvertZZ
sur le quel f est de classe C 1 ?
4. Calculer l’intégrale double I = yf (x, y) dxdy, où
D

D = {(x, y) ∈ R2 / x ≤ 0 et x2 + y 2 ≤ 1}.

42
S.M. DOUIRI

Solution.
Exercice 4.1.1
1. F, 2. F, 3. V,
4. F, 5. V, 6. F.
Exercice 4.1.2
1. Df = {(x, y) ∈ R2 / f (x, y) ∈ R} = R2 .
2. Étude de la fonction f sur R2 \{(1, 0)} :
2.1. Sur R2 \{(1, 0)}, la fonction f est une fraction rationnelle, alors elle est continue en
tout point où le dénominateur ne s’annule pas, c’est-à-dire qu’elle est continue sur
R2 \{(1, 0)}.
2.2. Sur R2 \{(1, 0)}, la fonction f est une fraction rationnelle, alors elle admet des déri-
S.
vées partielles premières sur R2 \{(1, 0)}. Pour tout (x, y) ∈ R2 \{(1, 0)}, on a
" #
∂f (x − 1)2 + y 2 − (x − 1) × 2(x − 1)
(x, y) = y 2
∂x ((x − 1)2 + y 2 )2
M
y − y (x − 1)2
4 2
= .
((x − 1)2 + y 2 )2
∂f 2y ((x − 1)2 + y 2 ) − y 2 × 2 y
(x, y) = (x − 1)
∂y ((x − 1)2 + y 2 )2
2y(x − 1) 3
.D
= .
((x − 1)2 + y 2 )2

∂f ∂f
2.3. Sur R2 \{(1, 0)}, les dérivées partielles premières et sont des fractions ration-
∂x ∂y
nelles, alors elles sont continues en tout point où le dénominateur ne s’annule pas,
c’est-à-dire qu’elles sont continues sur R2 \{(1, 0)}. Par suite f est de classe C 1 sur
R2 \{(1, 0)}.
O
2.4. La fonction f est de classe C 1 sur R2 \{(1, 0)}, alors elle est différentiable sur R2 \{(1, 0)}.
3. Étude de la fonction f en (1, 0) :
UI
3.1. En posant t = x − 1, on obtient

(x − 1)y 2 t y2
lim f (x, y) = lim = lim .
(x,y)→(1,0) (x,y)→(1,0) (x − 1)2 + y 2 (t,y)→(0,0) t2 + y 2

t y2 t y2
Or y 2 ≤ t2 +y 2 , ∀(t, y) ∈ R2 , alors Ce qui implique que lim = 0.
RI
≤ |t|.
t2 + y 2 (t,y)→(0,0) t2 + y 2
Par conséquence, lim f (x, y) = 0 = f (1, 0), c’est-à-dire, f est continue en (1, 0).
(x,y)→(1,0)

f (x, 0) − f (1, 0) 0−0


3.2. On a lim = lim = 0. Alors f possède une dérivée partielle
x→1 x−1 x→1 x−1
∂f
première en (1, 0) par rapport à la première variable x et on a (1, 0) = 0. De
∂x
f (1, y) − f (1, 0) 0−0
même, lim = lim = 0. Alors f possède une dérivée partielle
y→0 y−0 y→0 y−0
∂f
première en (1, 0) par rapport à la deuxième variable y et on a (1, 0) = 0.
∂y

43
S.M. DOUIRI

3.3. Pour étudier la différentiabilité de f en (1, 0), on calcule la limite suivante :


!
∂f ∂f
f (1 + x, y) − f (1, 0) − (1, 0)x + (1, 0)y
∂x ∂y
lim ε(x, y) = lim
(x,y)→(0,0) (x,y)→(0,0) k(x, y)k2
2
xy
= lim 3
(x,y)→(0,0) (x2 + y 2 ) 2

x3 1
Comme lim ε(x, y) = lim+ 3 = √ =6 0, alors f n’est pas différentiable
(x, y) → (0, 0) x→0 (2 x2 ) 2 2 2
x=y>0
en (1, 0).
S.
3.4. La fonction f n’est pas différentiable en (1, 0), alors elle n’est pas de classe C 1 sur
R2 et d’après la question 2.3, R2 \{(1, 0)} est le plus grand ouvert sur lequel f est de
classe C 1 .
4. On pose g(x, y) = y f (x, y). Remarquons que pour tout (x, y) ∈ D\{(1, 0)}, on a
M
(x − 1)(−y)2
g(x, −y) = −y f (x, −y) = −y = −y f (x, y) = −g(x, y) et (x, y) ∈ D
(x − 1)2 + (−y)2
si, et seulement si, (x, −y) ∈ D. Alors en effectuant un changement de variables via
l’application injective ϕ : R2 −→ R2 définie par ϕ(u, v) = (u, −v) = (x, y) on a D = ϕ(D)
et Jϕ (u, v) = −1. Alors
.D
ZZ ZZ ZZ
I= g(x, y) dxdy = g(x, y) dxdy = g ◦ ϕ(u, v)|Jϕ (u, v)| dudv
ZZD ϕ(D)
ZZ D

= g(u, −v) dudv = − g(u, v) dudv = −I.


D D

Ce qui entraine que I = 0.


O
4.2 Examen d’Analyse III (11 Mars 2020)
Exercice 4.2.1. Soient a ∈ Rn , f : Rn −→ R et g : R −→ Rn , x 7−→ (g1 (x), · · · , gn (x)) deux
UI
fonctions de plusieurs variables et A un ensemble de Rn . Dire si les affirmations suivantes sont
vraies V ou fausses F . (Bonne réponse≈ 1, Mauvaise réponse≈ −1 et absence de réponse≈ 0)
1. V Si A est borné, alors A est compact.
2. V Si A est compact et (xk )k est une suite de A, alors la suite (xk )k converge vers x et
x ∈ A.
RI
3. V Si f est continue sur Rn , alors f est différentiable en tout point de Rn .
4. V Si f et g sont différentiables, alors f ◦ g est dérivable sur R en t ∈ R et
n
X ∂f
(f ◦ g) (t) = (g(t))gi (t)
0 0

i=1 ∂xi
.
5. V Si g1 est différentiable en a, alors g est aussi différentiable.
6. V Si f ∈ C 2 (R2 ) et α et β sont les deux valeurs propres de la matrice hessienne de f en
a telles que αβ < 0, alors f admet un maximum local en a.

44
S.M. DOUIRI

7. V Si n = 2 et f (x, y) = ex−y (x2 − 2y 2 ), alors le gradient


    
∇f (x, y) = ex−y x2 − 2y 2 + 2x , ex−y −x2 + 2y 2 − y

Exercice 4.2.2. Justifier toutes les réponses.


Soit f la fonction définie par f (x, y) = xy 2 .
1. Montrer que la fonction f possède une infinité de points critiques.
2. Déterminer les extremums de f . Sont-ils globaux ?
3. Montrer que la condition f (x, y) = sin(xy) définit y comme fonction de x au voisinage de
(1, 0).
ZZ
4. Calculer l’intégrale double I = f (x, y) dxdy, où
S.
D

D = {(x, y) ∈ R2 / x ≥ 0, y ≥ 0 et x2 + y 2 ≤ 1}.

Solution.
M
Exercice 4.2.1
1. F, 2. F, 3, F, 4. V,
5. F, 6. F, 7. F.
Exercice 4.2.2
!
.D
∂f ∂f
1. (x, y) est un point critique de f si, et seulement si, ∇(f ) = (x, y), (x, y) = (0, 0).
∂x ∂y
C’est-à-dire, (x, y) est la solution du système

∂f


 (x, y) = y 2 = 0 (
y= 0
∂x ⇐⇒ ⇐⇒ x ∈ R et y = 0.



∂f
(x, y) = 2 x y = 0 y = 0 ou x = 0
∂y
O
Ce qui entraine que f admet une infinité de points critiques qui constituent la droite
∆ = {(x, y) ∈ R2 / y = 0}.
2. Déterminons d’abord la matrice Hessienne de f en chaque point (x, y). On a
UI
∂ 2f ∂ 2f ∂ 2f ∂ 2f
(x, y) = 0, (x, y) = 2 x et (x, y) = (x, y) = 2 y.
∂x2 ∂y 2 ∂x∂y ∂y∂x
!
0 2y
Alors Hf (x, y) = , d’où det Hf (x, y) = −4 y 2 . En particulier pour chaque point
2y 2x
RI
critique (a, 0) on a det Hf (a, 0) = 0, donc on ne peut conclure a priori. Essayons d’étudier
si la différence f (x, y) − f (a, 0) change de signe ou non au voisinage de (a, 0). Pour tout
(a, 0) ∈ ∆, on a f (x, y) − f (a, 0) = x y 2 .
F Si a > 0, il existe r > 0 tel que x y 2 ≥ 0, ∀(x, y) ∈]a − r, a + r[×] − r, r[, c’est-à-dire,
f (x, y) − f (a, 0) ≥ 0, ∀(x, y) ∈ Bk k∞ ((a, 0), r). Ce qui signifie que f possède un minimum
local en (a, 0).
F Si a < 0, il existe r > 0 tel que x y 2 ≤ 0, ∀(x, y) ∈]a − r, a + r[×] − r, r[, c’est-à-dire,
f (x, y) − f (a, 0) ≤ 0, ∀(x, y) ∈ Bk k∞ ((a, 0), r). Ce qui signifie que f admet un maximum
local en (a, 0).
F Si a = 0. Pour tout ε > 0, sur la boule Bk k∞ ((0, 0), ε) et suivant le chemin x = y, on a

45
S.M. DOUIRI

f (x, y) − f (0, 0) = f (x, x) = x3 qui change de signe selon x. Ce qui signifie que f n’admet
pas d’extremum local en (0, 0).
Lorsque f possède des extremums locaux en (a, 0) avec a 6= 0, on a f (a, 0) = 0. Or
f (1, 2) = 4 > f (a, 0) et f (−1, −2) = −4 < f (a, 0), alors ces extremums ne sont pas
globaux.
∂g
3. On pose g(x, y) = f (x, y)−sin(xy) = x y 2 −sin(xy), qui de classe C ∞ (R2 ), alors (x, y) =
∂y
∂g
2 x y − x cos(xy). donc g(1, 0) = 0 et (1, 0) = −1 6= 0. Le Théorème des fonctions
∂y
implicites entraine alors qu’il existe un voisinage U de 1, un voisinage V de 0 et une
fonction implicite ϕ : U ←− V de classe C ∞ (U ) tels que
i) U × V ⊂ R2 et ϕ(1) = 0;
S.
ii) Pour tout x ∈ U, on a g(x, y) = 0 ⇐⇒ y = ϕ(x). C’est-à-dire, la condition f (x, y) =
sin(x y) définit y comme fonction de x au voisinage de (1, 0).
∂g
(x, ϕ(x))
M
iii) ϕ (x) = −
0 ∂x , ∀x ∈ U.
∂g
(x, ϕ(x))
∂y
ZZ ZZ
4. Calculons l’intégrale double I = f (x, y) dxdy = x y 2 dxdy où
D D
D = {(x, y) ∈ R2 / x ≥ 0, y ≥ 0 et x2 + y 2 ≤ 1}. En effectuant un changement de
.D
variables en coordonnées polaires, via l’application injective de classe C 1 sur R∗+ × [0, 2π[
définie par
ψ : R∗+ × [0, 2π[ −→ R2
(r, θ) 7−→ ψ(r, θ) = (r cos θ, r sin θ) = (x, y)
on a Jψ (r, θ) = r et D = {(x, y) ∈ R2 / x ≥ 0, y ≥ 0 et x2 + y 2 ≤ 1} = ψ(D0 ), où

D0 = {(r, θ) ∈ R∗+ × [0, 2π[ / r cos θ ≥ 0, r sin θ ≥ 0 et r2 ≤ 1}


O
= {(r, θ) ∈ R∗+ × [0, 2π[ / cos θ ≥ 0, sin θ ≥ 0 et 0 < r ≤ 1}
π
= {(r, θ) ∈ R∗+ × [0, 2π[ / 0 < r ≤ 1 et 0 ≤ θ ≤ }.
2
UI
Alors ZZ ZZ
I= f (x, y) dxdy = x y 2 dxdy
ZZD ψ(D0 )

= r cos θ r2 sin2 θ r drdθ


ψ(D0 )
Z 1 Z π
4 2
= r dr cos θ sin2 θ dθ
RI
"0 #1 "0 #π
r5 sin3 θ 2
1 1 1
= × = × = .
5 0
3 0
5 3 15

4.3 Examen d’Analyse III (13 Janvier 2020)


Exercice 4.3.1. [6.5 points=1+1+1+0.5+1+1+1]Les quatre questions suivantes sont indé-
pendantes.
1. 1.1. Donner la définition d’un voisinage d’un point a ∈ Rn .
1.2. Montrer que l’intersection finie de voisinages de a est un voisinage de a.

46
2. Montrer que l’ensemble ∆ = {(x, y) ∈ R2 / x ≤ 0 et 0 ≤ y ≤ ex } est un fermé de R2 .
Est-il compact ?
3. 3.1. Donner la définition de la différentiabilité d’une fonction numérique f : Rn → R
en un point a ∈ Rn .
3.2. En utilisant cette définition, prouver que la fonction réelle d’une seule variable f :
x 7→ sin x est différentiable en a ∈ R et donner sa différentielle en a.
4. Donner la définition d’un point critique d’une fonction différentiable f : Rn → R.
Exercice 4.3.2. [9.5 points=0.5+1+2+1.5+1+1+0.5+1+1]
Soit g la fonction définie par g(x, y) = xy ln(x2 + y 2 ).
1. Quel est le domaine de définition de la fonction g.
2. Montrer que g possède un prolongement par continuité sur R2 défini par
S.
(
xy ln(x2 + y 2 ) si (x, y) 6= (0, 0) ;
f (x, y) =
0 sinon.
(Indication : Pour tout α > 0, on a lim+ tα ln t = 0.)
M
t→0
3. Etudier l’existence des dérivées partielles premières de f sur R2 .
4. Etudier la différentiabilité de f en chaque point de R2 et donner sa différentielle lorsqu’elle
existe.
.D
5. Déterminer le plus grand ouvert de R2 sur lequel f est de classe C 1 .
6. 6.1. Montrer que l’égalité

f (x, y) = 0 définit implicitement y comme fonction de x au
1 3
voisinage de ( 2 , 2 ).
6.2. Donner un développement limité à l’ordre 1 de cette fonction au voisinage de 12 .
7. 7.1. Montrer que l’ensemble D = {(x, y) ∈ R2 / y ≥ 0 et x2 + y 2 ≤ 1} est convexe.
ZZ
7.2. Calculer l’intégrale double I = f (x, y) dxdy.
O
D

Exercice 4.3.3. [3 points=1+1+1]


Soit f la fonction définie sur R2 par f (x, y) = x2 (y 2 + ex ).
1. Montrer que (a, b) est un point critique de f si, et seulement si,
UI
(a, b) ∈ {0} × R ∪ {(−2, 0)}.
2. Soit (a, b) un point critique de f .
2.1. Si (a, b) ∈ {0} × R, vérifier que f admet un minimum global en (a, b).
2.2. Si (a, b) = (−2, 0), la fonction f possède-elle un extremum en (−2.0)?
RI
Solution.
Exercice 4.3.1
1. 1.1. Définition d’un voisinage d’un point a ∈ Rn : On dit qu’une partie V de Rn est un
voisinage de a si V contient une boule de centre a.
1.2. Montrons que l’intersection finie de voisinages de a est aussi un voisinage de a :
Soient V1 , V2 , . . . , Vk , des voisinages de a, alors il existe ri > 0 avec i = 1, 2, . . . , k
tels que B(a, ri ) ⊂ Vi (1). Comme l’ensemble {i = 1, 2, · · · k} est fini, alors si on pose
r = min{ri | i = 1, 2, . . . , k}, on trouve que r > 0. Or r ≤ ri , ∀i ∈ {i = 1, 2, . . . , k},
ce qui implique que B(a, r) ⊂ B(a, ri ), ainsi avec (1), on a B(a, r) ⊂ Vi ∀i ∈ {i =
1, 2, . . . , k}, donc B(a, r) ⊂ ∩i=k i=1 Vi . D’où l’intersection ∩i=1 Vi est un voisinage de a.
i=k

47
2. Montrons que l’ensemble ∆ = {(x, y) ∈ R2 / x ≤ 0 et 0 ≤ y ≤ ex } est un fermé de
R2 : Soit (Wn )n∈N une suite de ∆qui converge vers `. alors Wn = (xn , yn ) avec xn ≤ 0 et
0 ≤ yn ≤ exn (2), ` = (x, y) ∈ R2 , lim(xn ) = x et lim(yn ) = y. Par passage à la limite
dans (2), on trouve que x ≤ 0 et 0 ≤ y ≤ ex , c’est-àdire que ` = (x, y) ∈ ∆. On conclut
alors que ∆ est un fermé de R2 .
La présentation graphique suivante de ∆, montre que ∆ est non borné.
S.
M
Sinon, alors il existe un r > 0 tel que ∆ ⊂ Bk k∞ ((0, 0), r). Comme r > −2, alors −r − 1 <
1 = e0, donc (0, −r − 1) ∈ ∆, ce qui donne que c (0, −r − 1) ∈ Bk k∞ ((0, 0), r), c’est-à-dire
que k(0, −r − 1)k∞ = r + 1 < r. Absurde. Alors ∆ est non compact.
3. 3.1. Définition de la différentiabilité d’une fonction numérique f : Rn → R en un point
.D
a ∈ Rn : On dit que f est différentiable en a, s’il existe une forme linéaire L sur Rn
telle que pour tout h ∈ Rn on a

f (a + h) = f (a) + L(h) + khkε(h) avec lim ε(h) = 0.


h→0

3.2. D’après le développement limité de la fonction h 7→ sin(a + h) en 0, on a


O
sin(a + h) = sin(a) + h cos(a) + khkε(h) avec lim ε(h) = 0.
h→0

Il suffit de poser L(h) = h cos(a), donc L est forme linéaire sur R. Par suite la
fonction sin est différentiable sur R.
UI
4. On dit que x0 est un point critique d’une fonction numérique différentiable f : Rn → R,
si df (x0 ) = 0.
Exercice 4.3.2
Soit g la fonction définie par g(x, y) = xy ln(x2 + y 2 ).
1. Soit Dg le domainde de définition de la fonction g. Alors (x, y) ∈ Dg si et seulement si
RI
x2 + y 2 > 0, c’est équivalent à (x, y) 6= (0, 0). D’où Dg = R2 \(0, 0).
2. Montrons que g possède un prolongement par continuité : on a

lim |g(x, y)| = lim xy ln(x2 + y 2 )


(x,y)→(0,0) (x,y)→(0,0)

= lim |r2 cos(θ) sin(θ) ln(r2 )| avec x = r cos(θ) et y = r sin(θ)


r→0
= lim |2r2 cos(θ) sin(θ) ln(r)|
r→0
≤ lim |2r2 ln(r)| = | lim 2r2 ln(r)|, car | cos(θ) sin(θ)| ≤ 1.
r→0 r→0
| {z }
=0

48
Donc lim g(x, y) = 0. Ainsi g admet un prolongement par continuité sur R2 défini
(x,y)→(0,0)
par (
xy ln(x2 + y 2 ) si (x, y) 6= (0, 0) ;
f (x, y) =
0 sinon.

3. Étudions l’existence des dérivées partielles premières de f sur R2 : Soient y ∈ R et


fy : x 7→ xy ln(x2 + y 2 ) la première fonction partielle de f . Comme fy = f1 × f2 ◦ f3 avec
f1 : x 7→ xy, f2 : t 7→ ln(t) et f3 : x 7→ x2 +y 2 sont des fonctions dérivables respectivement
sur R, R+∗ et R et f3 (R∗ ) ⊂ R+∗ , alors fy est dérivable sur R∗ et sur R si y 6= 0. Donc
∂f
existe sur R2 \(0, 0). De plus on a
∂x
∂f 2x
S.
(x, y) = fy0 (x) = y ln(x2 + y 2 ) + xy · 2
∂x x + y2
2x y
2
= y ln(x2 + y 2 ) + 2 .
x + y2
M
De même si (x, y) 6= (0, 0), nous trouvons que
∂f 2y
(x, y) = x ln(x2 + y 2 ) + xy · 2
∂y x + y2
2y 2 x
= x ln(x2 + y 2 ) + 2
.D
.
x + y2
si (x, y) = (0, 0), nous étudions les limites suivantes :

f (x, 0) − f (0, 0) 0
lim = lim = 0
x→0 x−0 x→0 x
f (0, x) − f (0, 0) 0
lim = lim = 0.
O
x→0 x−0 x→0 x
∂f ∂f
Ce qui est équivalent à (0, 0) = (0, 0) = 0.
∂x ∂y
4. Étudions la différentiabilité de f en chaque point de R2 et donnons sa différentielle :
UI
• Sur R2 \(0, 0) on a f = f4 × f5 ◦ f6 avec f4 : (x, y) 7→ xy, f5 : t 7→ ln(t) et f6 : (x, y) 7→
x2 + y 2 sont des fonctions différentiable respectivement sur R2 , R+∗ et R2 ; et puisque
f6 (R2 \(0, 0)) ⊂ R+∗ , alors f est différentiable sur R2 \(0, 0).
• Si (x, y) = (0, 0), nous étudions la limite suivante :
RI
∂f ∂f
f (x, y) − f (0, 0) − (0, 0)x − (0, 0)y
∂x ∂y
lim ε(x, y) = lim
(x,y)→(0,0) (x,y)→(0,0) k(x, y)k2
xy ln(x2 + y 2 )
= lim √ 2
(x,y)→(0,0) x + y2
r2 cos(θ) sin(θ) ln(r2 )
= lim √ avec x = r cos(θ) et y = r sin(θ).
r→0 r2
= lim 2r cos(θ) sin(θ) ln(r) = 0,
r→0

49
car la fonction θ 7→ cos(θ) sin(θ) est bornée et lim r ln(r) = 0. Alors f est différentiable
r→0
en (0, 0). En résumant ce qui précède, nous trouvons que pour tout h = (h1 , h2 ) ∈ R2 , on
a df(x,y) (h) =

 2x2 y 2y 2 x

y ln(x2 + y 2 ) + · h1 + x ln(x 2
+ y 2
) + · h2 si (x, y) 6= (0, 0) ;
 x2 + y 2 x2 + y 2
 0 sinon.

5. Déterminons le plus grand ouvert de R2 sur lequel f est de classe C 1 : D’après la question
3 on a

 2x2 y
y ln(x2 + y 2 ) + 2

si (x, y) 6= (0, 0) ;
S.
∂f
(x, y) = x + y2
∂x 
 0 sinon.
2x2 y
Comme l’application (x, y) 7→ y ln(x2 + y 2 ) + est continue sur R2 \{(0, 0)}, alors
x2 + y 2
M
∂f
il suffit d’étudier la continuité de en (0, 0). Posons x = r cos(θ) et y = r sin(θ). Alors
∂x

∂f 2 2 2x2 y
lim (x, y) = lim y ln(x + y ) + 2
.D
(x,y)→(0,0) ∂x (x,y)→(0,0) x + y2
2 2(r cos(θ))2 r sin(θ)
= lim r sin(θ) ln(r ) +
r→0 r2
= lim 2r sin(θ) ln(r) + 2r cos(θ))2 sin(θ)
r→0
∂f
=0= (0, 0),
∂x
O
car les fonctions θ 7→ cos(θ)2 sin(θ) et θ 7→ sin(θ) sont bornées, lim r = 0 et lim r ln(r) = 0.
r→0 r→0
∂f ∂f
Donc est continue en (0, 0). On Déduit que est continue sur R2 . De la même façon
∂x ∂x
UI
∂f
nous montrons que est continue sur R2 . Ce qui entraîne que f est de classe C 1 sur R2 .
∂y
6. 6.1. Montrons que l’égalité f (x, y) = 0 définit implicitement y comme √ fonction de x
√ √ 3
au voisinage de ( 21 , 23 ) : On a f est de classe C k et f ( 21 , 23 ) = ln(1) = 0 et
4
∂f 1 √3 3
RI
( 2 , 2 ) = 6= 0, alors d’après le théorème des fonctions implicites, il existe un
∂y 4 √
voisinage I de 12 et un voisinage J de 23 et une fonction implicite ϕ : I → J de
classe C k tels que

— ϕ( 21 ) = 2
3
;
— f (x, y) = 0 ⇔ y = ϕ(x) ∀ x ∈ I ;
∂f
(x, ϕ(x))
— ∀ x ∈ I on ϕ0 (x) = − ∂x .
∂f
(x, ϕ(x))
∂y

50
1
6.2. Donnons le développement limité à l’ordre 1 de ϕ au voisinage de 2
: Puisque ϕ est
de classe C 1 , alors
1 1 1 1
ϕ(x) = ϕ( ) + (x − )ϕ0 ( ) + (x − )ε(x).
2 2 2 2

∂f 1 √3 3 √
(2, 2 ) 3
0 1
Avec lim1 ε(x) = 0. Or on a ϕ ( 2 ) = − ∂x 4
=− 3 =− . D’où
∂f 1 3√
3
( , )
x→ 2
∂y 2 2 4
√ √
3 3 1 1
ϕ(x) = − (x − ) + (x − )ε(x).
2 3 2 2
S.
Avec lim1 ε(x) = 0
x→ 2

7. 7.1. Montrons que l’ensemble D = {(x, y) ∈ R2 / y ≥ 0 et x2 + y 2 ≤ 1} est convexe : on


a
M
D = {(x, y) ∈ R2 / x2 + y 2 ≤ 1} ∩ {(x, y) ∈ R2 / y ≥ 0}
q
= {(x, y) ∈ R2 / x2 + y 2 ≤ 1} ∩ {(x, y) ∈ R2 / y ≥ 0}
= {(x, y) ∈ R2 / k(x, y)k2 ≤ 1} ∩ {(x, y) ∈ R2 / y ≥ 0}
.D
= Bk·k2 ((0, 0), 1) ∩ {(x, y) ∈ R2 / y ≥ 0}
| {z }
:=C

Soient a = (a1 , a2 ) et b = (b1 , b2 ) deux éléments de D, alors a, b ∈ Bk·k2 ((0, 0), 1)


et a, b ∈ C. Comme les boules fermés sont convexes, donc pour montrer que D est
convexe, il suffit de prouver que [a, b] ⊂ C. Soit z = (z1 , z2 ) ∈ [a, b], alors il existe
O
t ∈ [0, 1] tel que z = ta + (1 − t)b, ainsi z2 = ta + (1 − t)b2 , or t, 1 − t, a2 et b2 sont
des nombres positive, d’où z2 ≥ 0, par suite z ∈ C. C’est-à-dire que [a, b] ⊂ C.
ZZ
7.2. Calculons l’intégrale double I = f (x, y) dxdy : On utilise le changement de
UI
D
variables x = u et y = −v. La matrice jacobienne de ce changement de variables est
donc : !
1 0
.
0 −1
Le déterminant jacobien vaut donc -1. Comme (x, y) ∈ D ⇐⇒ (u, v) ∈ D et on a
RI
par la formule du changement de variables :
ZZ ZZ
I= f (x, y) dxdy = f (u, −v) dudv.
D D

Or f (u, −v) = −f (u, v), donc


ZZ ZZ
I= f (u, −v) dudv = − f (u, v) dudv = −I.
D D

Ce qui donne que 2I = 0, par suite I = 0. Nous pouvons aussi utiliser les cordonnées
polaires pour montrer que I = 0

51
Exercice 4.3.3
Soit f la fonction définie sur R2 par f (x, y) = x2 (y 2 + ex ).
1. (a, b) est un point critique de f si, et seulement si,

∂f


 (a, b) = 0 (
2a(b2 + ea ) + a2 ea = 0
∂x ⇔
 ∂f

 (a, b) = 0 a2 × 2b = 0.
∂y
C’équivalent à
( ( (
2a(b2 + ea ) + a2 ea = 0 a=0 b=0
⇔ ou
a = 0 ou b = 0 b∈R 2aea + a2 ea = 0.
S.
(
b=0
⇔ (a, b) ∈ {0} × R ou ⇔ (a, b) ∈ {0} × R ou (a, b) = (−2, 0).
2a + a2 = 0.
M
2. Soit (a, b) un point critique de f .
2.1. Si (a, b) ∈ {0}×R,, alors f (a, b) = 0. Or f (x, y) = x2 (y 2 +ex ) ≥ 0 = f (a, b), ∀(x, y) ∈
R2 . Donc f admet un minimum global en (a, b).
2.2. Si (a, b) = (−2, 0). On a
∂ 2f
.D
• (x, y) = (2x + 2)(y 2 + ex ) + (x2 + 2x)ex
∂x2
∂ 2f ∂ 2f ∂ 2f
• (x, y) = 2x 2
et (x, y) = (x, y) = 4xy. Alors
∂y 2 ∂x∂y ∂y∂x
!
(2x + 2)(y 2 + ex ) + (x2 + 2x)ex 4xy
Hf (x, y) = .
4xy 2x2
O
−2e−2 0
Ce qui implique que det Hf (−2, 0) = = −16e−2 < 0, donc la fonction f
0 8
ne possède pas d’extremum en (−2, 0), mais seulement un point selle.
UI
4.4 Examen d’Analyse III (07 Janvier 2019)
Exercice 4.4.1.
1. Soit k · k une norme sur Rn . Montrer que
RI
|kxk − kyk| ≤ kx − yk, ∀x, y ∈ Rn .

2. Soit B(a, r) une boule ouverte de Rn . Prouver que B(a, r) est un ouvert convexe.
3. Soit f : D ⊂ Rn −→ Rp est fonction définie sur un ouvert D par f (x) = (f1 (x), f2 (x), . . . , fp (x)).
Soient a ∈ Rn et l = (l1 , l2 , . . . , lp ) ∈ Rp . Montrer que lim f (x) = l si et seulement si
x→a
lim
x→a
f j (x) = lj ∀j ∈ {1, . . . , p}.

4. Énoncer le théorème des fonctions implicites pour une fonction numérique définie sur un
ouvert de R2 .

52
Exercice 4.4.2. Soient m ∈ N et f la fonction définie sur R2 par :

 xm y
 si (x, y) 6= (0, 0);
f (x, y) = x2 + y 2

 0 si (x, y) = (0, 0).

1. Montrer que f est continue sur D = R2 \{(0, 0)} pour tout m ∈ N.


2. Calculer les dérivées partielles de f en chaque point de D et pour tout m ∈ N.
3. Justifier pourquoi f est de classe C 1 sur D et pour tout m ∈ N.
4. La fonction f est-elle différentiable sur D ?
5. Dans cette question, nous étudions f en (0, 0).
S.
5.1. Étudier suivant m la continuité de f en (0, 0).
5.2. Étudier l’existence des dérivées partielles de f en (0, 0) pour tout m ∈ N.
5.3. Pour quelles valeurs de m, f est-elle différentiable en (0, 0) ?
M
5.4. Selon les valeurs de m, déterminer le plus grand ouvert de R2 sur lequel f est de
classe C 1 .
6. Pour m = 0, on considère la fonction g : R −→ R2 définie par g(t) = (t, −t). Montrer que
g ◦ f est différentiable sur D et calculer la matrice jacobienne de g ◦ f .
.D
7. Pour m = 1, on pose
ZZ
I= f (x, y) dxdy avec ∆ = {(x, y) ∈ R2 : x2 + y 2 − y < 0}.

7.1. Dessiner ∆ et montrer que ∆ est un ouvert de R2 .


7.2. Calculer I.
O
Exercice 4.4.3. Afin de traiter une infection bactérienne, l’utilisation conjointe de deux com-
posés chimiques est proposée. Des études ont montré que la durée de l’infection pouvait être
modélisée par
f (x, y) = x2 + 2y 2 − 18x − 24y + 2xy + 120,
UI
où x est le dosage en mg du premier composé et y est le dosage en mg du second. Comment
minimiser la durée de l’infection ?

Solution.
Exercice 4.3.1
RI
1. Pour tous x et y de Rn on a kxk = kx − y + yk ≤ kx − yk + kyk, alors

kxk − kyk ≤ kx − yk. (4.1)

De même, on a kyk = ky − x + xk ≤ kx − yk + kxk, alors kyk − kx − yk ≤ kxk, c’est-à-dire,

− kx − yk ≤ kxk − kyk. (4.2)

D’après (4.1) et (4.2), on déduit que

|kxk − kyk| ≤ kx − yk, ∀x, y ∈ Rn .

53
2. Soit B(a, r) = {x ∈ Rn /kx−ak < r} une boule ouverte de Rn . Pour tout x ∈ B(a, r), on a
kx−ak < r, c’est-à-dire, r −kx−ak > 0. Choisissons alors 0 < ε < r −kx−ak et montrons
que B(x, ε) ⊂ B(a, r). En effet, pour tout y ∈ B(x, ε) on a ky − xk < ε < r − kx − ak,
alors ky − xk + kx − ak < r. D’où, ky − ak = ky − x + x − ak ≤ ky − xk + kx − ak < r.
C’est-à-dire, y ∈ B(a, r) et par suite, la boule ouverte B(a, r) est un ouvert de Rn .
Pour la convexité de B(a, r), soient x et y deux éléments de B(a, r). Pour tout z ∈ [x, y],
il existe t ∈ [0, 1] tel que z = x + t (y − x). Alors on a
kz − ak = kx + t (y − x) − ak = kx + t (y − a + a − x) − ak
= kt (y − a) + (1 − t)(x − a)k
≤ t k(y − a)k + (1 − t) k(x − a)k
< t r + (1 − t) r
S.
kz − ak < r.
Ce qui implique que z ∈ B(a, r), d’où [x, y] ⊂ B(a, r).
3. Supposons que lim f (x) = l. Soit j ∈ {1, . . . , p} et pour tout ε > 0 il existe η > 0 tel que
x→a
si kx − ak < η on a kf (x) − lk∞ < ε (puisque toutes les normes sont équivalentes dans
M
Rp , on peut utiliser la norme sup), ce qui implique que |fj (x) − lj | ≤ kf (x) − lk∞ < ε.
D’où lim fj (x) = lj .
x→a
Réciproquement, pour tout ε > 0 il existe ηj > 0 tel que kx − ak < ηj implique que
|fj (x) − lj | < ε. En choisissant η = min ηj on obtient l’implication
.D
j∈{1,...,p}

kx − ak < η =⇒ kx − ak < ηj =⇒ |fj (x) − lj | < ε, ∀j ∈ {1, . . . , p} =⇒ kf (x) − lk∞ < ε.


Par conséquence, lim f (x) = l.
x→a
4. Soient U un ouvert de R2 , (a, b) ∈ U et f : U −→ R une fonction de classe C k sur U. Si
f (a, b) = 0 et ∂f
∂y
(a, b) 6= 0, alors il existe un voisinage I de a, un voisinage J de b (I et J
sont des intervalles ouverts contenant a et b respectivement) et une fonction ϕ : I −→ J
O
de classe C k sur I vérifiant
i) I × J ⊂ U et ϕ(a) = b;
ii) f (x, ϕ(x)) = 0, ∀x ∈ I, (ou l’équivalence suivante : ∀x ∈ I, f (x, y) = 0 ⇐⇒ y =
ϕ(x)).
UI
∂f
(x, ϕ(x))
iii) ϕ0 (x) = − ∂f
∂x
.
∂y
(x, ϕ(x))
Exercice 4.3.2
Soient m ∈ N et f une fonction qui dépend du paramètre m. Plus précisément, pour tout
m ∈ N∗ on a
RI
 m
 x y

si (x, y) 6= (0, 0);
f (x, y) = x2 + y 2

 0 si (x, y) = (0, 0)
et si m = 0, on a  y
 si (x, y) 6= (0, 0);
f (x, y) = + y2
x2

0 si (x, y) = (0, 0).
1. Montrons que f est continue sur D = R2 \{(0, 0)} pour tout m ∈ N.
Sur D, la fonction f est une fraction rationnelle bien définie. Donc elle est continue sur
D.

54
2. Calculons les dérivées partielles de f en chaque point de D et pour tout m ∈ N.
 y
 , si (x, y) 6= (0, 0);
Pour m = 0 : On a f (x, y) = x + y2
2

0, si (x, y) = (0, 0).
Soit (x, y) ∈ D, c’est-à-dire, (x, y) 6= (0, 0). La fonction f possède-t-elle, d’abord, des dérivées
partielles au point (x, y)? Notons par f1 : R −→ R et f2 : R −→ R les deux fonctions partielles
de f en (x, y).
Pour f1 :
— Si y = 0 on a f1 (t) = 0, ∀t ∈ R, alors f1 est dérivable en x et f10 (x) = 0.
y
— Si y 6= 0 on a f1 (t) = 2 , ∀t ∈ R, alors f1 est une fraction rationnelle qui est dérivable
t + y2
−2xy
S.
en x et f10 (x) = 2 .
(x + y 2 )2
Par suite, f admet au point (x, y) une dérivée partielle par rapport à la 1ère variable x et on a 1

∂f −2xy
M
(x, y) = f10 (x) = 2 , ∀(x, y) ∈ D.
∂x (x + y 2 )2

Pour f2 :

1

, si t 6= 0; −1
.D
— Si x = 0 on a f2 : t 7−→ alors f2 est dérivable en y ∈ R∗ et f20 (y) = 2 .
 t
0, si t = 0, y
t
— Si x 6= 0 on a f2 : t 7−→ 2 , alors f2 est une fraction rationnelle qui est dérivable en
x + t2
x2 − y 2
y et f20 (y) = 2 .
(x + y 2 )2
Par suite, f admet au point (x, y) une dérivée partielle par rapport à la 2ème variable y et on
O
a2
∂f x2 + y 2 − 2y 2 x2 − y 2
(x, y) = f20 (y) = = , ∀(x, y) ∈ D.
∂y (x2 + y 2 )2 (x2 + y 2 )2
UI
y
Autrement, sur D la fonction f est une fraction rationnelle ”f (x, y) = 2 ”, alors elle admet
x + y2
des dérivées partielles par rapport aux deux variables x et y. Donc pour tout (x, y) ∈ D on a

∂f −2xy
(x, y) = 2 ,
∂x (x + y 2 )2
RI
et
∂f x2 + y 2 − 2y 2 x2 − y 2
(x, y) = = .
∂y (x2 + y 2 )2 (x2 + y 2 )2
Pour m ∈ N∗ : On a 
 xm y
 , si (x, y) 6= (0, 0);
f (x, y) = x 2 + y2

 0, si (x, y) = (0, 0).

1. Cette formule est valable pour les deux cas y = 0 et y 6= 0.


2. Cette formule est valable pour les deux cas x = 0 et x 6= 0.

55
xm y
Sur D la fonction f est une fraction rationnelle ”f (x, y) = ”, alors elle admet des
x2 + y 2
dérivées partielles par rapport aux deux variables x et y. Donc pour tout (x, y) ∈ D on a

∂f mxm−1 y(x2 + y 2 ) − xm y × 2x xm−1 y((m − 2)x2 + my 2 )


(x, y) = = ,
∂x (x2 + y 2 )2 (x2 + y 2 )2

et
∂f xm (x2 + y 2 ) − xm y × 2y xm (x2 − y 2 )
(x, y) = = .
∂y (x2 + y 2 )2 (x2 + y 2 )2
Par l’étude de la dérivabilité des fonctions partielles : Soit (x, y) ∈ D, c’est-à-dire, (x, y) 6= (0, 0).
La fonction f possède-t-elle des dérivées partielles au point (x, y)? Notons par f1 : R −→ R et
f2 : R −→ R les deux fonctions partielles de f en (x, y).
S.
Pour f1 :
— Si y = 0 on a f1 (t) = 0, ∀t ∈ R, alors f1 est dérivable en x et f10 (x) = 0.
tm y
— Si y 6= 0 on a f1 (t) = 2 , ∀t ∈ R, alors f1 est une fraction rationnelle qui est dérivable
M
t + y2
xm−1 y((m − 2)x2 + my 2 )
en x et f10 (x) = .
(x2 + y 2 )2
Par suite, f admet au point (x, y) une dérivée partielle par rapport à la 1ère variable x et on a 3
.D
∂f xm−1 y((m − 2)x2 + my 2 )
(x, y) = f10 (x) = , ∀(x, y) ∈ D.
∂x (x2 + y 2 )2

Pour f2 :
— Si x = 0 on a f2 (t) = 0, ∀t ∈ R, alors f2 est dérivable en y et f20 (y) = 0.
xm t
— Si x 6= 0 on a f2 (t) = 2 , ∀t ∈ R, alors f2 est une fraction rationnelle qui est dérivable
x + t2
O
xm (x2 + y 2 ) − xm y × 2y xm (x2 − y 2 )
en y et f20 (y) = = .
(x2 + y 2 )2 (x2 + y 2 )2
Par suite, f admet au point (x, y) une dérivée partielle par rapport à la 2ème variable y et on
UI
a4
∂f xm (x2 − y 2 )
(x, y) = f20 (y) = , ∀(x, y) ∈ D.
∂y (x2 + y 2 )2
∂f −2xy ∂f
3. ? Pour m = 0 : Sur D on a les deux dérivées partielles (x, y) = 2 et (x, y) =
∂x (x + y )
2 2 ∂y
x2 − y 2
RI
sont des fractions rationnelles dont leurs dénominateurs ne s’annulent pas, alors
(x2 + y 2 )2
elles sont continues sur D. D’où la fonction f est de classe C 1 sur l’ouvert D.
∂f xm−1 y((m − 2)x2 + my 2 )
? Pour m ∈ N : Sur D on a les deux dérivées partielles

(x, y) = et
∂x (x2 + y 2 )2
∂f xm (x2 − y 2 )
(x, y) = sont des fractions rationnelles dont leurs dénominateurs ne s’annulent
∂y (x2 + y 2 )2
pas, alors elles sont continues sur D. D’où la fonction f est de classe C 1 sur l’ouvert D.
3. Cette formule est valable pour les deux cas y = 0 et y 6= 0.
4. Cette formule est valable pour les deux cas x = 0 et x 6= 0.

56
4. f est de classe C 1 sur l’ouvert D, alors elle est différentiable sur D.
5. Étudions la fonction f au point (0, 0).
(a) Pour la continuité de f en (0, 0) :
 y
 , si (x, y) 6= (0, 0);
Pour m = 0 : f (x, y) = x + y2
2

0, si (x, y) = (0, 0).
Cherchons, la limite de f au point (0, 0) suivant des chemins particuliers passant par
(0, 0). Par exemple, selon le chemin y = 0, on a
lim f (x, y) = lim y
x2 +y 2
(x, y) → (0, 0) (x, y) → (0, 0)
y=0 y=0
= lim 02 = 0.
S.
x→0 x

Suivant le chemin x = 0 on a
lim f (x, y) = lim y
x2 +y 2
(x, y) → (0, 0) (x, y) → (0, 0)
M
x=0 x=0
= lim 1 = ±∞
y→0 y

La limite suivant ce chemin n’existe pas, ce qui entraîne que f n’est pas continue en (0, 0).
 m
 x y , si (x, y) 6= (0, 0);

.D
Pour m ∈ N∗ : f (x, y) = x2 + y 2

 0, si (x, y) = (0, 0).
x2 1
— Si m = 1 : lim f (x, y) = lim f (x, x) = lim 2x 2 = 6= f (0, 0). Alors f n’est pas
(x, y) → (0, 0) x→0 x→0 2
x=y
continue en (0, 0).
x2
O
— Si m ≥ 2 : On sait que ≤ 1, ∀(x, y) ∈ R2 \{(0, 0)}. Donc
x2 + y 2
xm y x2 × xm−2 y x2 |xm−2 y|
|f (x, y)| = = = ≤ |xm−2 y|
x +y
2 2 x +y
2 2 x +y
2 2
UI
Or, lim xm−2 y = 0, alors lim f (x, y) = 0 = f (0, 0). C’est-à-dire, f est continue
(x,y)→(0,0) (x,y)→(0,0)
en (0, 0), ∀m ≥ 2.
Autrement, en utilisant les coordonnées polaires x = r cos θ et y = r sin θ, on obtient
rm cosm θ.r sin θ
RI
|f (x, y) − f (0, 0)| = = |rm−1 cosm θ. sin θ|
r2
= |rm−1 | · | cosm θ sin θ|
≤ |rm−1 |.

Comme lim |rm−1 | = 0, alors lim f (x, y) = f (0, 0). D’où f est continue en (0, 0), ∀m ≥ 2.
r→0 (x,y)→(0,0)

(b) Pour les dérivées partielles de f en (0, 0) :


 y
 , si (x, y) 6= (0, 0);
— Si m = 0 : f (x, y) = x + y2
2

0, si (x, y) = (0, 0).

57
f (t, 0) − f (0, 0) 0−0
? lim = lim = 0. Alors f possède une dérivée partielle par rapport à x
t→0 t−0 t→0 t
∂f
en (0, 0) et on a (0, 0) = 0.
∂x
1
f (0, t) − f (0, 0) −0 1 1
? lim = lim t = lim 2 = + = +∞. Donc f n’admet pas une dérivée
t→0 t−0 t→0 t t→0 t 0
partielle par rapport à y en (0, 0).
 m
 x y , si (x, y) 6= (0, 0);

— Si m ≥ 1 : f (x, y) =  x2 + y 2
 0, si (x, y) = (0, 0).
f (t, 0) − f (0, 0) 0−0
? lim = lim = 0. Alors f possède une dérivée partielle par rapport à x
t−0
S.
t→0 t→0 t
∂f
en (0, 0) et on a (0, 0) = 0.
∂x
f (0, t) − f (0, 0) 0−0
? lim = lim = 0. Donc f admet une dérivée partielle par rapport à y en
t→0 t−0 t→0 t
M
∂f
(0, 0) et on a (0, 0) = 0.
∂y
(c) Étudions la différentiabilité de f en (0, 0) :
— Pour m = 0 : f n’admet pas une dérivée partielle par rapport à y en (0, 0), donc f
n’est pas différentiable en (0, 0).
.D
— Pour m = 1 : f n’est pas continue en (0, 0), donc f n’est pas différentiable en (0, 0).
— Pour m ≥ 2 : Étudions la limite en (0, 0) de l’application ε : R2 −→ R, définie par
!
∂f ∂f
f (x, y) − f (0, 0) − x. (0, 0) + y. (0, 0)
∂x ∂y
ε(x, y) = .
k(x, y)k
O
En utilisant la norme k k2 , on obtient
!
∂f ∂f
f (x, y) − f (0, 0) − x. (0, 0) + y. (0, 0)
∂x ∂y
ε(x, y) =
UI
k(x, y)k2
f (x, y) xm y xm y
= = 1 = 3 .
k(x, y)k2 (x2 + y 2 )(x2 + y 2 ) 2 (x2 + y 2 ) 2
x2 y
? Si m = 2 : ε(x, y) = 3 et suivant le chemin x = y, on a
(x2 + y 2 ) 2
RI
x2 y
lim ε(x, y) = lim 3
(x, y) → (0, 0) (x, y) → (0, 0) (x2 + y 2 ) 2
x=y x=y
x3
1
= lim 3 = 3 6= 0.
x→0 (2x2 ) 2 22
Alors, f n’est pas différentiable en (0, 0) 5 .
xm y
? Si m ≥ 3 : ε(x, y) = 3 et en utilisant les coordonnées polaires x = r cos θ et y =
(x2 + y 2 ) 2
5. C’est un contre exemple de f continue ; f différentiable.

58
r sin θ, avec r > 0 et θ ∈ [0, 2π[, on obtient

xm y
lim ε(x, y) = lim 3
(x,y)→(0,0) (x,y)→(0,0) (x2 + y 2 ) 2

rm cosm θr sin θ
lim ε(x, y) = lim 3
(x,y)→(0,0) r→0 (r2 ) 2
θ ∈ [0, 2π[
= lim rm−2 cosm θ sin θ
r→0
θ ∈ [0, 2π[
= 0
car lim rm−2 = 0 et −1 ≤ cosm θ sin θ ≤ 1, ∀θ ∈ [0, 2π[. Par conséquence, la fonction f est
S.
r→0
différentiable en (0, 0) pour tout m ≥ 3.
(d) D’après la question 3), la fonction f est de classe C 1 sur l’ouvert 6 D.
? Pour m ∈ {0, 1, 2} : f n’est pas différentiable en (0, 0), donc f n’est pas de classe C 1 sur
M
R2 . Alors D est de plus grand ouvert de R2 sur lequel f est de classe C 1 .
? Pour m ≥ 3 : La fonction f est de classe C 1 sur l’ouvert D. Reste à étudier la continuité des
dérivées partielles en (0, 0). On a

xm−1 y((m − 2)x2 + my 2 )
.D

∂f  si (x, y) 6= (0, 0);
(x, y) = (x2 + y 2 )2
∂x 

0 si (x, y) = (0, 0).

et 
xm (x2 − y 2 )

∂f  si (x, y) 6= (0, 0);
(x, y) = (x 2 + y 2 )2
∂y 

0 si (x, y) = (0, 0).
O
? En utilisant les coordonnées polaires x = r cos θ et y = r sin θ, avec r > 0 et θ ∈ [0, 2π[, on
obtient
UI
∂f xm−1 y((m − 2)x2 + my 2 )
lim (x, y) = lim
(x,y)→(0,0) ∂x (x,y)→(0,0) (x2 + y 2 )2
rm−1 cosm−1 θ · r sin θ((m − 2)r2 cos2 θ + mr2 sin2 θ)
= lim
r→0 r4
θ ∈ [0, 2π[
h i
RI
= lim rm−2 cosm−1 θ sin θ((m − 2) cos2 θ + m sin2 θ)
r→0
θ ∈ [0, 2π[
∂f ∂f
lim (x, y) = 0 = (0, 0),
(x,y)→(0,0) ∂x ∂x

∂f
car |cosm−1 θ sin θ((m − 2) cos2 θ + m sin2 θ)| ≤ 2m − 2 et lim rm−2 = 0. Donc est continue
r→0 ∂x
en (0, 0).

6. D est un ouvert de R2 , car Dc = {(0, 0)} est un fermé de R2 .

59
De même, on a

∂f xm (x2 − y 2 )
lim (x, y) = lim
(x,y)→(0,0) ∂y (x,y)→(0,0) (x2 + y 2 )2
rm cosm θ.(r2 cos2 θ − r2 sin2 θ)
= lim
r→0 r4
θ ∈ [0, 2π[
= lim rm−2 cosm θ.(r2 cos2 θ − r2 sin2 θ)
r→0
θ ∈ [0, 2π[
∂f ∂f
lim (x, y) = 0 = (0, 0),
(x,y)→(0,0) ∂y ∂y
S.
∂f
car |cosm θ.(cos2 θ − sin2 θ)| = |cosm θ. cos(2θ)| ≤ 1 et lim rm−2 = 0. Donc est continue en
r→0 ∂y
(0, 0).
M
∂f ∂f
Par conséquence, les dérivées partielles et sont continues sur R2 , ce qui signifie que la
∂x ∂y
fonction f est de classe C 1 sur le plus grand ouvert R2 .

6. Pour m = 0, on considère la fonction g : R −→ R2 définie par g(t) = (t, −t). Montrer que
.D
g ◦ f est différentiable sur D et calculer la matrice jacobienne de g ◦ f .
La fonction vectorielle g : R −→ R2 est différentiable sur R, car g1 : t 7→ t et g2 : t 7→ −t sont
différentiables (dérivables) sur R. Or, la fonction numérique f est différentiable sur D, alors la
fonction composée g ◦ f est différentiable sur D.
Soit (x, y) ∈ D, on a
 
O
∂g1 !
 (f (x, y)) ∂f ∂f
Jg◦f (x, y) = Jg (f (x, y)).Jf (x, y) =  ∂t
 ∂g2
.
 (x, y), (x, y)
(f (x, y)) ∂x ∂y
! ∂t ! ! !
g10 (f (x, y)) ∂f ∂f 1 ∂f ∂f
UI
= 0 . (x, y) (x, y) = . (x, y) (x, y)
g2 (f (x, y)) ∂x ∂y −1 ∂x ∂y
   
∂f ∂f −2xy x2 − y 2
 (x, y) (x, y)   2 
 (x + y 2 )2 (x2 + y 2 )2 
=  ∂x ∂y
 
= 
 ∂f ∂f   2xy y 2 − x2 
− (x, y) − (x, y)
∂x ∂y (x2 + y 2 )2 (x2 + y 2 )2
RI
Autrement, on a g ◦ f (x, y) = g(f (x, y)) = (f (x, y), −f (x, y)), alors
 
∂f ∂f
 (x, y) (x, y) 
Jg◦f (x, y) = 

∂x ∂y 

 ∂f ∂f 
− (x, y) − (x, y)
∂x ∂y

Exercice 4.3.3
La fonction f (x, y) = x2 + 2y 2 − 18x − 24y + 2xy + 120 représente la durée de l’infection en
mélangeant le dosage x en mg du premier composé et le dosage y en mg du second composé.

60
Déterminons d’abord les points critiques de la fonction f qui est de classe C ∞ (R2 ).

∂f


 (x, y) = 0 (
2x − 18 + 2y = 0
df (x, y) = 0L(R2 ,R) ⇔ ∂x ⇔



∂f
(x, y) = 0 4y − 24 + 2x = 0
∂y
( ( (
x−9+y =0 y−3=0 y=3
⇔ ⇔ ⇔
2y − 12 + x = 0 x−9+y =0 x=9−y =9−3=6
La fonction f possède alors un seul point critique (6, 3).

Pour la matrice Hessienne Hf (x, y) on a


S.
∂ 2f ∂ 2f ∂ 2f ∂ 2f
(x, y) = 2, (x, y) = 4 et (x, y) = (x, y) = 2.
∂x2 ∂y 2 ∂x∂y ∂y∂x
Donc !
M
2 2
Hf (x, y) = .
2 4
On a det(Hf (6, 3)) = 8 − 4 = 4 > 0 et 2 > 0, alors f admet un minimum local en (6, 3). Est-il
global ? En effet, pour tout (x, y) ∈ R2 , on a
.D
f (6 + x, 3 + y) − f (6, 3) = (6 + x)2 + 2(3 + y)2 − 18(6 + x) − 24(3 + y) + 2(6 + x)(3 + y) + 120 − 30
= x2 + 2xy + y 2 = (x + y)2 ≥ 0.

Par suite, on obtiendra une durée minimale de l’infection en utilisant un dosage de 6mg du
premier composé et 3mg du second.
O
UI
RI

61

Vous aimerez peut-être aussi